Discussion:
Hütchen-Spiel
(zu alt für eine Antwort)
Rainer Rosenthal
2020-03-26 11:06:54 UTC
Permalink
Weil die 1 bei der Umordnung 2, 3, 4, ..., 1 von 1, 2, 3, 4, ...
auf einem Platz mit unendlichem Index landet.
Wo ist denn eine solche Umordnung beschrieben?

Wir haben es mit zwei Ordnungen der natürlichen Zahlen zu tun:

Ord_A = [1, 2, 3, 4, ...]

und

Ord_Z = [2, 3, 4, ..., 1]


Dabei ist Ord_A uns allen wohlbekannt, und über Ord_Z hatte ich mich mit
Dir unterhalten können. Dabei bekam ich sogar Zustimmung von Dir, dass
in Ord_Z gilt: das kleinste (niederste) Element in {1,2,3} ist die 2.

Danach ging es leider nicht mehr weiter, und ich habe auf Anraten kluger
Leute pausiert. Dabei wollte ich neulich bereits erfahren, was oben steht:

Wo ist denn eine Abfolge von Umordnungen beschrieben, die von Ord_A zu
Ord_Z führen kann? Wer behauptet denn, dass das überhaupt möglich sei?

Wohin auch immer ich "die 1 defilieren lasse", startend bei Ord_A, komme
ich stets nur zu Ordnungen der Form

Ord_M = [2, 3, 4, ... , 1, ...]

Dein Hütchen-Spiel-Vergleich ist treffend. Angeblich hast Du Cantor
dabei erwischt. Aber wo? Du scheinst ihm eher vorzuwerfen, dass er dies
Hütchenspiel nicht sauber hinbekommen würde.

Gruß,
Rainer Rosenthal
***@web.de
Ganzhinterseher
2020-03-26 15:45:47 UTC
Permalink
Post by Rainer Rosenthal
Weil die 1 bei der Umordnung 2, 3, 4, ..., 1 von 1, 2, 3, 4, ...
auf einem Platz mit unendlichem Index landet.
Wo ist denn eine solche Umordnung beschrieben?
Oben.
Post by Rainer Rosenthal
Ord_A = [1, 2, 3, 4, ...]
und
Ord_Z = [2, 3, 4, ..., 1]
Dabei ist Ord_A uns allen wohlbekannt, und über Ord_Z hatte ich mich mit
Dir unterhalten können. Dabei bekam ich sogar Zustimmung von Dir, dass
in Ord_Z gilt: das kleinste (niederste) Element in {1,2,3} ist die 2.
Natürlich. Da besteht kein Dissens.
Post by Rainer Rosenthal
Danach ging es leider nicht mehr weiter, und ich habe auf Anraten kluger
Wo ist denn eine Abfolge von Umordnungen beschrieben, die von Ord_A zu
Ord_Z führen kann? Wer behauptet denn, dass das überhaupt möglich sei?
Ich behaupte, dass jede Ordnung der natürlichen Zahlen möglich ist, WENN alle so vollständig existieren dass eine surjektive Abbildung in sie möglich ist und WENN alle endlich sind und auf endlichen Plätzen sitzen. Was sollte unter diesen Umstanden jeder beliebigen Ordnung entgegenstehen?
Post by Rainer Rosenthal
Wohin auch immer ich "die 1 defilieren lasse", startend bei Ord_A, komme
ich stets nur zu Ordnungen der Form
Ord_M = [2, 3, 4, ... , 1, ...]
Das liegt daran, dass nur solche Plätze definierbar sind. Denn für alle definierbaren natürlichen Zahlen gilt

∀n ∈ ℕ_def: |ℕ \ {1, 2, 3, ..., n}| = ℵo.

Also bleiben stets unendlich viele übrig, die offenbar nicht definierbar sind (sonst würde sie ja aufgebraucht), aber trotzdem natürliche Zahlen sind.
Post by Rainer Rosenthal
Dein Hütchen-Spiel-Vergleich ist treffend. Angeblich hast Du Cantor
dabei erwischt. Aber wo? Du scheinst ihm eher vorzuwerfen, dass er dies
Hütchenspiel nicht sauber hinbekommen würde.
Wo? Schau Dein Beispiel an: Ord_M = [2, 3, 4, ... , 1, ...]
Oder meines: ∀n ∈ ℕ_def: |ℕ \ {1, 2, 3, ..., n}| = ℵo.

Cantor behauptet aber bis zum Ende zu kommen. Das sagt er zwar nicht so deutlich, aber er behauptet dass bei seiner surjektiven Abbildung keine Zahl übrig bleibt. Die Rechtfertigung nimmt er aus der Eigenschaft, dass "der aus unsrer Regel resultierende Zuordnungsprozeß keinen Stillstand" leidet. [Cantor, p. 239]

Diese Eigenschaft zeigt aber keineswegs Surjektivität. Denn in

Ord_M = [2, 3, 4, ... , 1, ...]

leidet die Tatsache, dass die 1 niemals näher ans Ziel kommt, auch keinen Stillstand. Es kommt eben immer darauf an, was man ins Auge fasst.

Kurz: Wenn die behauptete Surjektivität angenommen wird, dann ist jede Umordnung möglich. Dann aber sind dunkle Zahlen erforderlich, denn dann ist auch 2, 3, 4, ..., 1 möglich, aber die 1 sitzt nicht auf einem definierbaren Platz.

Gruß, WM
Juergen Ilse
2020-03-26 16:22:05 UTC
Permalink
Hallo,
Post by Ganzhinterseher
Ich behaupte, dass jede Ordnung der natürlichen Zahlen möglich ist, WENN alle so vollständig existieren dass eine surjektive Abbildung in sie möglich ist und WENN alle endlich sind und auf endlichen Plätzen sitzen.
Statt Behauptungen waeren *BEWEISE* aussagekraeftiger, aber Beweise fuer
IHRER Meinung nach "offensichtliche" Aussagen haben SIE ja hier in der
Gruppe noch nie vernueftig hinbekommen ...
Post by Ganzhinterseher
Das liegt daran, dass nur solche Plätze definierbar sind. Denn für alle definierbaren natürlichen Zahlen gilt
∀n ∈ ℕ_def: |ℕ \ {1, 2, 3, ..., n}| = ℵo.
Also bleiben stets unendlich viele übrig, die offenbar nicht definierbar sind (sonst würde sie ja aufgebraucht), aber trotzdem natürliche Zahlen sind.
Schon wieder dieser hahnebuechene Unsinn mit "definierbaren Plaetzen" ...
Jede solche Argumentation ist komplett fuer die Tonne, auch wenn SIE es nicht
wahrhaben wollen.

Tschuess,
Juergen Ilse (***@usenet-verwaltung.de)
Ganzhinterseher
2020-03-26 17:00:14 UTC
Permalink
Post by Juergen Ilse
Post by Ganzhinterseher
Das liegt daran, dass nur solche Plätze definierbar sind. Denn für alle definierbaren natürlichen Zahlen gilt
∀n ∈ ℕ_def: |ℕ \ {1, 2, 3, ..., n}| = ℵo. (*)
Also bleiben stets unendlich viele übrig, die offenbar nicht definierbar sind (sonst würde sie ja aufgebraucht), aber trotzdem natürliche Zahlen sind.
Schon wieder dieser hahnebuechene Unsinn mit "definierbaren Plaetzen" ...
Es ist keine Behauptung, sondern ein Beweis. Wären alle natürlichen Zahlen definierbar, dann brauchte man doch nicht unendlich viele übrig zu lassen. Warum sollte man das tun?

Gruß, WM
Juergen Ilse
2020-03-26 21:40:19 UTC
Permalink
Hallo,
Post by Ganzhinterseher
Post by Juergen Ilse
Schon wieder dieser hahnebuechene Unsinn mit "definierbaren Plaetzen" ...
Es ist keine Behauptung, sondern ein Beweis.
Ein Beweis ist kein duemmliches Geblubber mit seltsamen Begriffen, sondern
das zurueckfuehren der zu beweisenden Aussage auf Axiome oder bereits be-
wiesene Aussagen. So etwas findet sich in IHREM sinnlosen Gefasel noch
nicht einmal ansatzweise ...
Post by Ganzhinterseher
Wären alle natürlichen Zahlen definierbar, dann brauchte man doch nicht
unendlich viele übrig zu lassen. Warum sollte man das tun?
Warum SIE so etwas tun, weiss ich nicht. Warum solch intellektuellen
Sondermuell in dieser Newsgroup absondern, ist mir ebenfalls nicht
bekannt.

Tschuess,
Juergen Ilse (***@usenet-verwaltung.de)
Me
2020-03-26 17:12:41 UTC
Permalink
Post by Ganzhinterseher
er behauptet dass bei seiner surjektiven Abbildung keine Zahl übrig bleibt.
Das haben surjektive Abbildungen von irgendeiner Menge auf IN so an sich, sie mathematischer Pfeifenkopf.
Ganzhinterseher
2020-03-26 17:21:53 UTC
Permalink
Post by Me
Post by Ganzhinterseher
er behauptet dass bei seiner surjektiven Abbildung keine Zahl übrig bleibt.
Das haben surjektive Abbildungen von irgendeiner Menge auf IN so an sich
Richtig. Weshalb aber sollte eine natürliche Zahl von der Umordnungsmöglichkeit ausgeschlossen sein, hinter alle Zahlen, die es außer ihr gibt, eingeordnet zu werden. Alle, die es gibt, ist doch keine verbotene Bezeichnung in einer Theorie, in der alles gleichzeitig nebeneinander existiert und nicht erst nach und nach in Existenz gebracht wird.

Gruß, WM
Michael Klemm
2020-03-26 17:23:49 UTC
Permalink
Post by Ganzhinterseher
Post by Rainer Rosenthal
Weil die 1 bei der Umordnung 2, 3, 4, ..., 1 von 1, 2, 3, 4, ...
auf einem Platz mit unendlichem Index landet.
Wo ist denn eine solche Umordnung beschrieben?
Oben.
Ord_A = [1, 2, 3, 4, ...]
Post by Ganzhinterseher
Post by Rainer Rosenthal
und
Ord_Z = [2, 3, 4, ..., 1]
Post by Ganzhinterseher
Post by Rainer Rosenthal
Dabei ist Ord_A uns allen wohlbekannt, und über Ord_Z hatte ich mich mit
Dir unterhalten können. Dabei bekam ich sogar Zustimmung von Dir, dass
in Ord_Z gilt: das kleinste (niederste) Element in {1,2,3} ist die 2.
Natürlich. Da besteht kein Dissens.
Post by Rainer Rosenthal
Danach ging es leider nicht mehr weiter, und ich habe auf Anraten kluger
Wo ist denn eine Abfolge von Umordnungen beschrieben, die von Ord_A zu
Ord_Z führen kann? Wer behauptet denn, dass das überhaupt möglich sei?
Ich behaupte, dass jede Ordnung der natürlichen Zahlen möglich ist, WENN alle so vollständig existieren dass eine surjektive Abbildung in sie möglich ist und WENN alle endlich sind und auf endlichen Plätzen sitzen. Was sollte unter diesen Umstanden jeder beliebigen Ordnung entgegenstehen?
Die beiden Ordnungen sind nicht ordnungsisomorph, es gibt also keine Bijektion f: Ord_A -> Ord_Z mit x <_A y genau wenn x <_Z y für alle x, y e |N. Das hat Cantor ausführlich behandelt.

Gruß
Michael
Post by Ganzhinterseher
Post by Rainer Rosenthal
Wohin auch immer ich "die 1 defilieren lasse", startend bei Ord_A, komme
ich stets nur zu Ordnungen der Form
Ord_M = [2, 3, 4, ... , 1, ...]
Das liegt daran, dass nur solche Plätze definierbar sind. Denn für alle definierbaren natürlichen Zahlen gilt
∀n ∈ ℕ_def: |ℕ \ {1, 2, 3, ..., n}| = ℵo.
Also bleiben stets unendlich viele übrig, die offenbar nicht definierbar sind (sonst würde sie ja aufgebraucht), aber trotzdem natürliche Zahlen sind.
Post by Rainer Rosenthal
Dein Hütchen-Spiel-Vergleich ist treffend. Angeblich hast Du Cantor
dabei erwischt. Aber wo? Du scheinst ihm eher vorzuwerfen, dass er dies
Hütchenspiel nicht sauber hinbekommen würde.
Wo? Schau Dein Beispiel an: Ord_M = [2, 3, 4, ... , 1, ...]
Oder meines: ∀n ∈ ℕ_def: |ℕ \ {1, 2, 3, ..., n}| = ℵo.
Cantor behauptet aber bis zum Ende zu kommen. Das sagt er zwar nicht so deutlich, aber er behauptet dass bei seiner surjektiven Abbildung keine Zahl übrig bleibt. Die Rechtfertigung nimmt er aus der Eigenschaft, dass "der aus unsrer Regel resultierende Zuordnungsprozeß keinen Stillstand" leidet. [Cantor, p. 239]
Diese Eigenschaft zeigt aber keineswegs Surjektivität. Denn in
Ord_M = [2, 3, 4, ... , 1, ...]
leidet die Tatsache, dass die 1 niemals näher ans Ziel kommt, auch keinen Stillstand. Es kommt eben immer darauf an, was man ins Auge fasst.
Kurz: Wenn die behauptete Surjektivität angenommen wird, dann ist jede Umordnung möglich. Dann aber sind dunkle Zahlen erforderlich, denn dann ist auch 2, 3, 4, ..., 1 möglich, aber die 1 sitzt nicht auf einem definierbaren Platz.
Gruß, WM
Ganzhinterseher
2020-03-26 19:24:17 UTC
Permalink
Post by Rainer Rosenthal
Post by Ganzhinterseher
Post by Rainer Rosenthal
Weil die 1 bei der Umordnung 2, 3, 4, ..., 1 von 1, 2, 3, 4, ...
auf einem Platz mit unendlichem Index landet.
Wo ist denn eine solche Umordnung beschrieben?
Oben.
Ord_A = [1, 2, 3, 4, ...]
Post by Ganzhinterseher
Post by Rainer Rosenthal
und
Ord_Z = [2, 3, 4, ..., 1]
Post by Ganzhinterseher
Post by Rainer Rosenthal
Dabei ist Ord_A uns allen wohlbekannt, und über Ord_Z hatte ich mich mit
Dir unterhalten können. Dabei bekam ich sogar Zustimmung von Dir, dass
in Ord_Z gilt: das kleinste (niederste) Element in {1,2,3} ist die 2.
Natürlich. Da besteht kein Dissens.
Post by Rainer Rosenthal
Danach ging es leider nicht mehr weiter, und ich habe auf Anraten kluger
Wo ist denn eine Abfolge von Umordnungen beschrieben, die von Ord_A zu
Ord_Z führen kann? Wer behauptet denn, dass das überhaupt möglich sei?
Ich behaupte, dass jede Ordnung der natürlichen Zahlen möglich ist, WENN alle so vollständig existieren dass eine surjektive Abbildung in sie möglich ist und WENN alle endlich sind und auf endlichen Plätzen sitzen. Was sollte unter diesen Umstanden jeder beliebigen Ordnung entgegenstehen?
Die beiden Ordnungen sind nicht ordnungsisomorph, es gibt also keine Bijektion f: Ord_A -> Ord_Z mit x <_A y genau wenn x <_Z y für alle x, y e |N. Das hat Cantor ausführlich behandelt.
Er hat aber vergessen, zu behandeln, dass ausgehend von der Bijektion zwischen natürlichen Zahlen und Plätzen in 1, 2, 3, ... jede Umordnung möglich ist (weil nur endliche Zahlen und Plätze vorhanden sind) ohne eine unendliche Zahl oder einen unendlich indizierten Platz einzuführen.

Gruß, WM
Michael Klemm
2020-03-27 10:00:10 UTC
Permalink
Post by Ganzhinterseher
Post by Rainer Rosenthal
Post by Ganzhinterseher
Post by Rainer Rosenthal
Weil die 1 bei der Umordnung 2, 3, 4, ..., 1 von 1, 2, 3, 4, ...
auf einem Platz mit unendlichem Index landet.
Wo ist denn eine solche Umordnung beschrieben?
Oben.
Ord_A = [1, 2, 3, 4, ...]
Post by Ganzhinterseher
Post by Rainer Rosenthal
und
Ord_Z = [2, 3, 4, ..., 1]
Post by Ganzhinterseher
Post by Rainer Rosenthal
Dabei ist Ord_A uns allen wohlbekannt, und über Ord_Z hatte ich mich mit
Dir unterhalten können. Dabei bekam ich sogar Zustimmung von Dir, dass
in Ord_Z gilt: das kleinste (niederste) Element in {1,2,3} ist die 2.
Natürlich. Da besteht kein Dissens.
Post by Rainer Rosenthal
Danach ging es leider nicht mehr weiter, und ich habe auf Anraten kluger
Wo ist denn eine Abfolge von Umordnungen beschrieben, die von Ord_A zu
Ord_Z führen kann? Wer behauptet denn, dass das überhaupt möglich sei?
Ich behaupte, dass jede Ordnung der natürlichen Zahlen möglich ist, WENN alle so vollständig existieren dass eine surjektive Abbildung in sie möglich ist und WENN alle endlich sind und auf endlichen Plätzen sitzen. Was sollte unter diesen Umstanden jeder beliebigen Ordnung entgegenstehen?
Die beiden Ordnungen sind nicht ordnungsisomorph, es gibt also keine Bijektion f: Ord_A -> Ord_Z mit x <_A y genau wenn x <_Z y für alle x, y e |N. Das hat Cantor ausführlich behandelt.
Hier fehlt bei mir hinten das f, also "… genau wenn f(x) <_Z f(y) für alle x, y e |N". Dein gefordertes Produkt von Transpositionen, nämlich die Identität von |N als Produkt von 0 Transpositionen, gibt es. Freilich ist die Identität hier nicht ordnungserhaltend. Du kannst also nicht erwarten, dass unmittelbar vor der 1 in der neuen Ordnung eine Zahl steht. Z-kleiner sind vielmehr die Zahlen, die A-größer als 1 sind.

Gruß
Michael
Post by Ganzhinterseher
Er hat aber vergessen, zu behandeln, dass ausgehend von der Bijektion zwischen natürlichen Zahlen und Plätzen in 1, 2, 3, ... jede Umordnung möglich ist (weil nur endliche Zahlen und Plätze vorhanden sind) ohne eine unendliche Zahl oder einen unendlich indizierten Platz einzuführen.
Gruß, WM
Ganzhinterseher
2020-03-27 19:53:36 UTC
Permalink
Am Freitag, 27. März 2020 11:00:12 UTC+1 schrieb Michael Klemm:

Dein gefordertes Produkt von Transpositionen, nämlich die Identität von |N als Produkt von 0 Transpositionen, gibt es. Freilich ist die Identität hier nicht ordnungserhaltend. Du kannst also nicht erwarten, dass unmittelbar vor der 1 in der neuen Ordnung eine Zahl steht.

Was steht da wohl?

Wenn es sich nur um eine Umordnung durch unendlich viele Transpositionen handelt, dann steht dort eine Zahl. Wenn das nicht möglich ist, wenn also nicht alle Zahlen umgeordnet werden können, dann ist auch eine surjektive und injektive Abbildung dieser Zahlen nicht möglich.

Gruß, WM
Michael Klemm
2020-03-28 10:00:36 UTC
Permalink
Post by Michael Klemm
Dein gefordertes Produkt von Transpositionen, nämlich die Identität von |N als Produkt von 0 Transpositionen, gibt es. Freilich ist die Identität hier nicht ordnungserhaltend. Du kannst also nicht erwarten, dass unmittelbar vor der 1 in der neuen Ordnung eine Zahl steht.
Was steht da wohl?
Wenn es sich nur um eine Umordnung durch unendlich viele Transpositionen handelt, dann steht dort eine Zahl. Wenn das nicht möglich ist, wenn also nicht alle Zahlen umgeordnet werden können, dann ist auch eine surjektive und injektive Abbildung dieser Zahlen nicht möglich.
Gruß, WM
Da bringst Du was durcheinander. Nimm die Güteklassen
2b < 3b < 4b < … < 1a
Da ist nichts so gut wie 1a. Trotzdem gibt es unter den schlechteren Klassen keine beste.

Gruß
Michael
Ganzhinterseher
2020-03-28 18:37:42 UTC
Permalink
Post by Michael Klemm
Da bringst Du was durcheinander. Nimm die Güteklassen
2b < 3b < 4b < … < 1a
Da ist nichts so gut wie 1a. Trotzdem gibt es unter den schlechteren Klassen keine beste.
Dein Beispiel ist keines. Rainer hat die Ord_M definiert:

(2, 3, 4, ..., n, 1, n+1, ...)

Betrachtet man alle geordneten Mengen dieser Klasse, dann kann man alle definierbaren Zahlen vor die 1 ziehen. Es gibt keine definierbare Zahl, bei der das scheitern würde. Trotzdem bleiben fast alle Zahlen hinter der 1. Warum wohl?

Gruß, WM
Michael Klemm
2020-03-28 19:42:02 UTC
Permalink
Post by Ganzhinterseher
Post by Michael Klemm
Da bringst Du was durcheinander. Nimm die Güteklassen
2b < 3b < 4b < … < 1a
Da ist nichts so gut wie 1a. Trotzdem gibt es unter den schlechteren Klassen keine beste.
(2, 3, 4, ..., n, 1, n+1, ...)
Betrachtet man alle geordneten Mengen dieser Klasse, dann kann man alle definierbaren Zahlen vor die 1 ziehen. Es gibt keine definierbare Zahl, bei der das scheitern würde. Trotzdem bleiben fast alle Zahlen hinter der 1. Warum wohl?
Gruß, WM
2b < 3b < 4b < … < 1a ist in Rainers Schreibweise Ord_Z = [2,3,4,...,1], wobei ich die beiden voneinander unabhängigen Bereiche mit b und a bezeichnet und die Ordnung explizit mit "<" angeben habe. Die Unbegriffe "undefinierbar" bzw. "dunkel" habe ich selbstverständlich auch gestrichen.

Gruß
Michael
Ganzhinterseher
2020-03-28 21:25:57 UTC
Permalink
Post by Michael Klemm
2b < 3b < 4b < … < 1a ist in Rainers Schreibweise Ord_Z = [2,3,4,...,1], wobei ich die beiden voneinander unabhängigen Bereiche mit b und a bezeichnet und die Ordnung explizit mit "<" angeben habe. Die Unbegriffe "undefinierbar" bzw. "dunkel" habe ich selbstverständlich auch gestrichen.
Tja, du scheinst nicht zu begreifen, dass die Ordnung Z nicht ohne undefinierbare Zahlen aus der Ordnung A erzeugt werden kann und dass ohne die Erzeugbarkeit der Ordnung Z auch keine Surjektivität bei Bijektionen möglich ist.

Gruß, WM
Michael Klemm
2020-03-29 06:56:47 UTC
Permalink
Post by Ganzhinterseher
Post by Michael Klemm
2b < 3b < 4b < … < 1a ist in Rainers Schreibweise Ord_Z = [2,3,4,...,1], wobei ich die beiden voneinander unabhängigen Bereiche mit b und a bezeichnet und die Ordnung explizit mit "<" angeben habe. Die Unbegriffe "undefinierbar" bzw. "dunkel" habe ich selbstverständlich auch gestrichen.
Tja, du scheinst nicht zu begreifen, dass die Ordnung Z nicht ohne undefinierbare Zahlen aus der Ordnung A erzeugt werden kann und dass ohne die Erzeugbarkeit der Ordnung Z auch keine Surjektivität bei Bijektionen möglich ist.
Gruß, WM
Wer sagt denn, dass die Ordnung 2b < 3b < 4b < … < 1a aus einer anderen Ordnung erzeugt wird? Du darfst auch problemlos die Teilkette 2b < 3b < 4b < … als potentiell unendlich ansehen. Das ist so ähnlich wie bei der Hyperbel y = 1/x. Da gibt es zwei voneinander unabhängige Zweige, und den Kurvenverlauf kann man mit Hilfe der Differentialrechnung recht genau beschreiben, ohne auch nur ein einziges Paar (x,y) explizit anzugeben. Ein Rechenschieber für die Skizze genügt
vollkommen.

Gruß
Michael
Me
2020-03-29 16:46:28 UTC
Permalink
Post by Michael Klemm
Du darfst auch problemlos die Teilkette 2b < 3b < 4b < … als potentiell
unendlich ansehen.
Kannst Du mal erklären, was Du damit meinst? Wie genau unterscheidet sich eine potentiell unendliche Teilkette

2b < 3b < 4b < … [???]

von einer

(a) endlichen: 2b < 3b < 4b < .. <kb (mit k e IN, k > 4)

(b) unendlichen: 2b < 3b < 4b < …

Kannst Du das vielleicht mal kurz definieren, also den Begriff "potentiell unendliche Teilkette" im Gegensatz zu (a) bzw. (b).
Michael Klemm
2020-03-29 18:21:01 UTC
Permalink
Post by Me
Post by Michael Klemm
Du darfst auch problemlos die Teilkette 2b < 3b < 4b < … als potentiell
unendlich ansehen.
Kannst Du mal erklären, was Du damit meinst? Wie genau unterscheidet sich eine potentiell unendliche Teilkette
2b < 3b < 4b < … [???]
von einer
(a) endlichen: 2b < 3b < 4b < .. <kb (mit k e IN, k > 4)
(b) unendlichen: 2b < 3b < 4b < …
Kannst Du das vielleicht mal kurz definieren, also den Begriff "potentiell unendliche Teilkette" im Gegensatz zu (a) bzw. (b).
Das könnte vielleicht für Finitisten gelten, die die obere Schranke dynamisch anpassen oder wie WM diesbezüglich nichts genaues wissen?

Gruß
Michael
Me
2020-03-29 18:51:25 UTC
Permalink
Post by Michael Klemm
Das könnte vielleicht für Finitisten gelten, die die obere Schranke dynamisch
anpassen oder [...]
Aha? Haben wir hier ein "finitistisches System" als Grundlage? Welches?
Michael Klemm
2020-03-29 18:58:57 UTC
Permalink
Post by Me
Post by Michael Klemm
Das könnte vielleicht für Finitisten gelten, die die obere Schranke dynamisch
anpassen oder [...]
Aha? Haben wir hier ein "finitistisches System" als Grundlage? Welches?
Halt das adhoc erfundene.

Gruß
Michael
Me
2020-03-29 19:20:24 UTC
Permalink
Post by Michael Klemm
Post by Me
Aha? Haben wir hier ein "finitistisches System" als Grundlage? Welches?
Halt das adhoc erfundene.
Ok, ich frag' auch nicht mehr weiter. :-P

Aber WMs Gerede von "potentiell unendlichen Mengen" ist wirklich unerträglich.

Es ist *völlig unklar*, was das überhaupt sein soll. Der Begriff ist auch in der einschlägigen Literatur nicht dokumentiert. Kurz: Er macht keinen Sinn.

Scheißdreck halt.
Roalto
2020-03-29 20:17:41 UTC
Permalink
A
Post by Me
Post by Michael Klemm
Post by Me
Aha? Haben wir hier ein "finitistisches System" als Grundlage? Welches?
Halt das adhoc erfundene.
Ok, ich frag' auch nicht mehr weiter. :-P
Aber WMs Gerede von "potentiell unendlichen Mengen" ist wirklich unerträglich.
Ob "potentiell" oder "aktual" ist völlig ohne Belang für die heutige Mathematik.
Es ist eine philosophische Frage und spielte früher in den Diskussionen über die Mathematik eine Rolle.
95% aller Mathematiker sind Formalisten, der homöopathische Rest - Konstruktivisten, Finitisten, Ultrafinitisten und was sonst noch!

Für Formalisten ist wichtig, dass das zu Grunde liegende Axiomensystem widerspruchsfrei ist. Es sind bis dato keine Widersprüche gefunden worden!
WM mit seiner epochalen Dümmlichkeit glaubt nun Widersprüche gefunden zu haben.

Sein grosses Vorbild ist der Ultrafinitist Doron Zeilberger, dem hat er auch
seine Arbeiten präsentiert. Der meinte nur, er habe keine Lust das zu lesen, da es sich nur um Philosophie handelt.
Im Gegensatz zu WM hat Zeilberger was auf der Pfanne.
Post by Me
Es ist *völlig unklar*, was das überhaupt sein soll. Der Begriff ist auch in der einschlägigen Literatur nicht dokumentiert. Kurz: Er macht keinen Sinn.
Scheißdreck halt.
Ja, es ist Scheissdreck.

Viel Spass weiterhin
Roalto
Helmut Richter
2020-03-29 20:49:31 UTC
Permalink
Post by Roalto
Sein grosses Vorbild ist der Ultrafinitist Doron Zeilberger, dem hat er auch
seine Arbeiten präsentiert. Der meinte nur, er habe keine Lust das zu lesen,
Darin unterscheidet sich Zeilberger von dieser Newsgruppe, deren
Mitglieder WMs Ausführung nicht nur für lesenswert, sondern sogar für
beantwortenswert halten.
--
Helmut Richter
Ralf Bader
2020-03-29 21:08:54 UTC
Permalink
Post by Helmut Richter
Post by Roalto
Sein grosses Vorbild ist der Ultrafinitist Doron Zeilberger, dem hat er auch
seine Arbeiten präsentiert. Der meinte nur, er habe keine Lust das zu lesen,
Darin unterscheidet sich Zeilberger von dieser Newsgruppe, deren
Mitglieder WMs Ausführung nicht nur für lesenswert, sondern sogar für
beantwortenswert halten.
Auch das stellt sich durchaus anders dar, wenn man die Sache direkt und
nicht um drei Ecken herum betrachtet:
https://sites.math.rutgers.edu/~zeilberg/Opinion68.html
Zeilberger ist zweifellos ein sehr fähiger Kombinatoriker. Seine
Ansichten über die Grundlagen der Mathematik sind jedoch bestenfalls
kleinkariert.
Roalto
2020-03-29 21:25:46 UTC
Permalink
Post by Ralf Bader
Post by Helmut Richter
Post by Roalto
Sein grosses Vorbild ist der Ultrafinitist Doron Zeilberger, dem hat er auch
seine Arbeiten präsentiert. Der meinte nur, er habe keine Lust das zu lesen,
Darin unterscheidet sich Zeilberger von dieser Newsgruppe, deren
Mitglieder WMs Ausführung nicht nur für lesenswert, sondern sogar für
beantwortenswert halten.
Auch das stellt sich durchaus anders dar, wenn man die Sache direkt und
https://sites.math.rutgers.edu/~zeilberg/Opinion68.html
Zeilberger ist zweifellos ein sehr fähiger Kombinatoriker. Seine
Ansichten über die Grundlagen der Mathematik sind jedoch bestenfalls
kleinkariert.
Bist du sicher, dass du verstehst, was andere hier schreiben?
Was stellt sich durchaus anders dar?

Viel Spass weiterhin
Roalto
Me
2020-03-30 03:53:32 UTC
Permalink
Post by Roalto
Post by Ralf Bader
Auch das stellt sich durchaus anders dar, wenn man die Sache direkt und
https://sites.math.rutgers.edu/~zeilberg/Opinion68.html
Was stellt sich durchaus anders dar?
Sein grosses Vorbild ist der Ultrafinitist Doron Zeilberger, dem hat er auch
seine Arbeiten präsentiert. Der meinte nur, er habe keine Lust das zu lesen,
da es sich nur um Philosophie handelt.
Darin unterscheidet sich Zeilberger von dieser Newsgruppe, deren
Mitglieder WMs Ausführung nicht nur für lesenswert [halten], sondern <etc.>
Nun kann auch ich Ralf nur zustimmen, wenn er schreibt:

"[...] das stellt sich durchaus anders dar, wenn man die Sache direkt und
nicht um drei Ecken herum betrachtet".

Tatsächlich äußert sich Zeilberger speziell in "Opionion 68" wie folgt:

Added Dec. 6, 2008: I am very pleased and honored that Wolfgang Mueckenheim quoted this opinion in lecture XI of his fascinating course.

Added March 27, 2011: Read Wolfgang Mueckenheim's fascinating book ! I especially like the bottom of page 112 and the top of page 113, that prove, once and for all, that (at least) the actual infinity is pure nonsense.

"Read Wolfgang Mueckenheim's fascinating book!" passt m. E. irgendwie nicht zu Deiner Aussage/Behauptung "Der meinte nur, er habe keine Lust das zu lesen [...]".

Nun hat Zeilberger es aber nicht dabei belassen, sondern noch die folgende "höchst fragwürdige" Behauptung aufgestellt:

"I especially like the bottom of page 112 and the top of page 113, that prove, /once and for all/, that (at least) the actual infinity is pure nonsense."

Daraufhin hat er wohl etwas Gegenwind von Seiten seiner Kollegen zu spüren bekommen - denn im Kontext der *Mathematik* sollte man vorsichtig sein, mit dem Wort "Beweis" bzw. der Behauptung "hat bewiesen". --- Vermutlich hat man ihn nach diesem "Beweis" gefragt. :-)

Daraufhin hat er sich offenbar nicht anders zu helfen gewusst als folgende absurde "Erklärung" abzugeben (die in völligem WIDERSPRUCH zu obiger Aussage/Behauptung steht):

Clarification added Aug. 25, 2011: My endorsement of Wolfgang Mueckenheim's wonderful book is purely philosophical. I have no expertise, or interest, in checking any possible technical claims that he may have made.

Aber "Widersprüche" sehen derartige Typen n u r im Zusammenhang mit der Mengenlehre, NIE in dem Mist, den sie selbst verzapfen!

Hinweis: "endorsement" heißt "Befürwortung", "Zustimmung", "Billigung", etc.

Er meinte als KEINESWEGS, "er habe keine Lust das zu lesen, da es sich nur um Philosophie handelt", sondern er sagt hier explizit, dass seine "Befürwortung", "Zustimmung", "Billigung" in Bezug auf WMs Buch, *rein philosophisch* begründet sei. Zu "technischen" Behauptungen (also zu logisch-mathematischen Behauptungen) in WMs Buch wolle er sich aber nicht äußern: I have no expertise, or interest, in checking any possible technical claims that he may have made.

Ein feiger und m. E. intellektuell extrem unredlicher "Rückzieher", nachdem er kurz zuvor noch lautstark tropetet hatte: "the bottom of page 112 and the top of page 113 [...] prove, /once and for all/, that (at least) the actual infinity is pure nonsense."

Von einem *Mathematiker* sollte man eigentlich schon ein gewisses Interesse an den "technischen Aspekten" im Zusammenhang mit einem "Argument" erwarten können, das angeblich "/once and for all/ [proves] that (at least) the actual infinity is pure nonsense".

Naja...

Jedenfalls beim Lesen diverser Opinions dieses Mannes kann man sich des Eindrucks nicht verwehren, es mit "pure nonsense" zu tun zu haben.
Roalto
2020-03-30 13:02:16 UTC
Permalink
Post by Me
Post by Roalto
Post by Ralf Bader
Auch das stellt sich durchaus anders dar, wenn man die Sache direkt und
https://sites.math.rutgers.edu/~zeilberg/Opinion68.html
Was stellt sich durchaus anders dar?
Sein grosses Vorbild ist der Ultrafinitist Doron Zeilberger, dem hat er auch
seine Arbeiten präsentiert. Der meinte nur, er habe keine Lust das zu lesen,
da es sich nur um Philosophie handelt.
Darin unterscheidet sich Zeilberger von dieser Newsgruppe, deren
Mitglieder WMs Ausführung nicht nur für lesenswert [halten], sondern <etc.>
"[...] das stellt sich durchaus anders dar, wenn man die Sache direkt und
nicht um drei Ecken herum betrachtet".
Added Dec. 6, 2008: I am very pleased and honored that Wolfgang Mueckenheim quoted this opinion in lecture XI of his fascinating course.
Added March 27, 2011: Read Wolfgang Mueckenheim's fascinating book ! I especially like the bottom of page 112 and the top of page 113, that prove, once and for all, that (at least) the actual infinity is pure nonsense.
"Read Wolfgang Mueckenheim's fascinating book!" passt m. E. irgendwie nicht zu Deiner Aussage/Behauptung "Der meinte nur, er habe keine Lust das zu lesen [...]".
"I especially like the bottom of page 112 and the top of page 113, that prove, /once and for all/, that (at least) the actual infinity is pure nonsense."
Daraufhin hat er wohl etwas Gegenwind von Seiten seiner Kollegen zu spüren bekommen - denn im Kontext der *Mathematik* sollte man vorsichtig sein, mit dem Wort "Beweis" bzw. der Behauptung "hat bewiesen". --- Vermutlich hat man ihn nach diesem "Beweis" gefragt. :-)
Clarification added Aug. 25, 2011: My endorsement of Wolfgang Mueckenheim's wonderful book is purely philosophical. I have no expertise, or interest, in checking any possible technical claims that he may have made.
Aber "Widersprüche" sehen derartige Typen n u r im Zusammenhang mit der Mengenlehre, NIE in dem Mist, den sie selbst verzapfen!
Hinweis: "endorsement" heißt "Befürwortung", "Zustimmung", "Billigung", etc.
Er meinte als KEINESWEGS, "er habe keine Lust das zu lesen, da es sich nur um Philosophie handelt", sondern er sagt hier explizit, dass seine "Befürwortung", "Zustimmung", "Billigung" in Bezug auf WMs Buch, *rein philosophisch* begründet sei. Zu "technischen" Behauptungen (also zu logisch-mathematischen Behauptungen) in WMs Buch wolle er sich aber nicht äußern: I have no expertise, or interest, in checking any possible technical claims that he may have made.
Ein feiger und m. E. intellektuell extrem unredlicher "Rückzieher", nachdem er kurz zuvor noch lautstark tropetet hatte: "the bottom of page 112 and the top of page 113 [...] prove, /once and for all/, that (at least) the actual infinity is pure nonsense."
Von einem *Mathematiker* sollte man eigentlich schon ein gewisses Interesse an den "technischen Aspekten" im Zusammenhang mit einem "Argument" erwarten können, das angeblich "/once and for all/ [proves] that (at least) the actual infinity is pure nonsense".
Naja...
Jedenfalls beim Lesen diverser Opinions dieses Mannes kann man sich des Eindrucks nicht verwehren, es mit "pure nonsense" zu tun zu haben.
Meine Quelle liegt tief in sci.math, wenn ich mich richtig erinnere.
Such es dir raus.
Zeilberger, als ein Eitler, fühlte sich gebauchfidelt, dass er bei Mückenheim zitiert wurde. Im Übrigen sehe ich keinen wesentlichen Unterschied zwischen
seinen addenda und meinem Geschriebenen.

Viel Spass weiterhin
Roalto
Me
2020-03-30 13:12:21 UTC
Permalink
Post by Roalto
Meine Quelle liegt tief in sci.math, wenn ich mich richtig erinnere.
Such es dir raus.
Nicht so wichtig.
Post by Roalto
Zeilberger, als ein Eitler, fühlte sich gebauchfidelt, dass er bei Mückenheim
zitiert wurde.
Das wird wohl diese Stelle sein:

"Added Dec. 6, 2008: I am very pleased and honored that Wolfgang Mueckenheim quoted this opinion in lecture XI of his fascinating course."
Post by Roalto
Im Übrigen sehe ich keinen wesentlichen Unterschied zwischen
seinen addenda und meinem Geschriebenen.
Wie meinen? Außer dass er darin genau das GEGENTEIL dessen sagt, was Du ihm "angedichtet" hast? Da dann. Ja, hast Recht: Ist kein wesentlicher Unterschied. :-)

*Du* hattest wohl auch keine Lust, das von mir Geschriebene zu lesen. :-)

EOD
Roalto
2020-03-30 14:11:04 UTC
Permalink
Post by Me
Post by Roalto
Meine Quelle liegt tief in sci.math, wenn ich mich richtig erinnere.
Such es dir raus.
Nicht so wichtig.
Post by Roalto
Zeilberger, als ein Eitler, fühlte sich gebauchfidelt, dass er bei Mückenheim
zitiert wurde.
"Added Dec. 6, 2008: I am very pleased and honored that Wolfgang Mueckenheim quoted this opinion in lecture XI of his fascinating course."
Post by Roalto
Im Übrigen sehe ich keinen wesentlichen Unterschied zwischen
seinen addenda und meinem Geschriebenen.
Wie meinen? Außer dass er darin genau das GEGENTEIL dessen sagt, was Du ihm "angedichtet" hast? Da dann. Ja, hast Recht: Ist kein wesentlicher Unterschied. :-)
*Du* hattest wohl auch keine Lust, das von mir Geschriebene zu lesen. :-)
EOD
Na, endlich!!!

Viel Spass weiterhin
Roalto
Ganzhinterseher
2020-03-30 15:36:04 UTC
Permalink
Post by Me
Post by Roalto
Post by Ralf Bader
Auch das stellt sich durchaus anders dar, wenn man die Sache direkt und
https://sites.math.rutgers.edu/~zeilberg/Opinion68.html
Was stellt sich durchaus anders dar?
Sein grosses Vorbild ist der Ultrafinitist Doron Zeilberger, dem hat er auch
seine Arbeiten präsentiert. Der meinte nur, er habe keine Lust das zu lesen,
da es sich nur um Philosophie handelt.
Darin unterscheidet sich Zeilberger von dieser Newsgruppe, deren
Mitglieder WMs Ausführung nicht nur für lesenswert [halten], sondern <etc.>
"[...] das stellt sich durchaus anders dar, wenn man die Sache direkt und
nicht um drei Ecken herum betrachtet".
Added Dec. 6, 2008: I am very pleased and honored that Wolfgang Mueckenheim quoted this opinion in lecture XI of his fascinating course.
Added March 27, 2011: Read Wolfgang Mueckenheim's fascinating book ! I especially like the bottom of page 112 and the top of page 113, that prove, once and for all, that (at least) the actual infinity is pure nonsense.
"Read Wolfgang Mueckenheim's fascinating book!" passt m. E. irgendwie nicht zu Deiner Aussage/Behauptung "Der meinte nur, er habe keine Lust das zu lesen [...]".
"I especially like the bottom of page 112 and the top of page 113, that prove, /once and for all/, that (at least) the actual infinity is pure nonsense."
Das betrifft diesen Abschnitt:

Die Behauptung, dass ℕ aus der unendlichen Mengenvereinigung
{1} ∪ {2} ∪ {3} ∪ ... = ℕ resultiert, führt zu folgender Konsequenz: Geben wir die Zahl 1 in ein Reservoir, sodann die Zahl 2, sodann die Zahl 3 und fahren unbegrenzt so fort, indem wir z. B. die Geisterstunde nutzend um 0:00 Uhr beginnen, um 0:30 Uhr den zweiten Schritt ausführen, um 0:45 Uhr den
dritten usw., dann enthält das Reservoir nach unendlich vielen Schritten, Schlag 1:00 Uhr, den Grenzwert der Mengenfolge, die Menge aller natürlichen Zahlen.

Aber diese Behauptung hat auch eine andere Konsequenz: Geben wir um 0:00 Uhr die Zahlen 1 bis 10 in das Reservoir und entnehmen sogleich wieder die Zahl 1, um 0:30 Uhr geben wir die Zahlen 11 bis 20 in das Reservoir und entnehmen sogleich wieder die Zahl 2, um 0:45 Uhr geben wir die Zahlen 21 bis 30 in das Reservoir und entnehmen sogleich wieder die Zahl 3 usw., dann enthält das Reservoir nach unendlich vielen Schritten um 1:00 Uhr nichts, denn von jeder natürlichen Zahl kann der Zeitpunkt angegeben werden, zu dem sie wieder entnommen wurde.

Die letztere Überlegung ist offensichtlich falsch, denn die Anzahl der Zahlen im Reservoir wächst in jedem Schritt um 9 und nimmt niemals ab. Und was noch deutlicher den Fehler aufzeigt, ist dies: Würden wir statt der Zahlen 1, 2, 3, ... die Zahlen 10, 20, 30, ... entnehmen, so enthielte das Reservoir um 1:00 Uhr unendlich viele Zahlen, nämlich alle nicht durch 10 teilbaren natürlichen Zahlen. Das Ergebnis eines Prozesses darf aber nicht von Bezeichnungswillkür
abhängen. Das gilt in der Wissenschaft und sogar im Märchen, wie das folgende
Bei-Spiel zeigt: Es waren einmal sechs Schwestern, die verliefen sich in einen dunklen Wald. Dort fand sie ein böser, alter Zauberer, der die Mädchen mästen wollte, um sie dann zu fressen.
Post by Me
Daraufhin hat er wohl etwas Gegenwind von Seiten seiner Kollegen zu spüren bekommen - denn im Kontext der *Mathematik* sollte man vorsichtig sein, mit dem Wort "Beweis" bzw. der Behauptung "hat bewiesen". --- Vermutlich hat man ihn nach diesem "Beweis" gefragt. :-)
Der Beweis ist die Absurdität des Ergebnisses, später unter dem Stichwort MacDuck bekanntgeworden (früher bereits unter Tristram Shandy von Fraenkel popularisiert, aber von modernen Matheologen totgeschwiegen).
Post by Me
Clarification added Aug. 25, 2011: My endorsement of Wolfgang Mueckenheim's wonderful book is purely philosophical. I have no expertise, or interest, in checking any possible technical claims that he may have made.
Er meinte also KEINESWEGS, "er habe keine Lust das zu lesen, da es sich nur um Philosophie handelt", sondern er sagt hier explizit, dass seine "Befürwortung", "Zustimmung", "Billigung" in Bezug auf WMs Buch, *rein philosophisch* begründet sei. Zu "technischen" Behauptungen (also zu logisch-mathematischen Behauptungen) in WMs Buch wolle er sich aber nicht äußern: I have no expertise, or interest, in checking any possible technical claims that he may have made.
Ein feiger und m. E. intellektuell extrem unredlicher "Rückzieher", nachdem er kurz zuvor noch lautstark trompetet hatte: "the bottom of page 112 and the top of page 113 [...] prove, /once and for all/, that (at least) the actual infinity is pure nonsense."
Nun immerhin hat er trotz widerwärtigster Pressionen seitens der Matheologen seinen ersten Satz stehengelassen und ihn nicht gelöscht. Höchst anerkennenswert. Dass er tatsächlich keine Ahnung hat, wird niemand glauben.

Inzwischen wurde der Beweis bekanntlich verfeinert (also für jeden denkbaren Leser unwiderlegbar gestaltet):

Die Menge der Endsegmente, die in

E(1) ∩ E(2) ∩ E(3) ∩ ... = { }

ohne Änderung des Ergebnisses fortfallen können, ist die Menge aller definierbaren Endsegmente. Wer behauptet, dass auch nur ein definierbares Endsegment dort stehenbleiben müsste, ist offenbar nicht sehr clever - to say the least. Die Menge der benötigten definierbaren Endsegmente ist leer. Beweis: Man definiere eines, das nicht entfallen kann.

Gruß, WM
Me
2020-03-30 18:29:45 UTC
Permalink
Setzen wir hier einmal voraus, dass es sinnvoll ist, sogenannte Supertasks zu betrachten. (Hinweis: Die Mengenlehre setzt derartige "Supertasks" nicht voraus. Vielmehr erlaubt erst die ML sich auf einer rationalen Basis mit derartigen Supertasks zu beschäftigen.)
Post by Ganzhinterseher
Die Behauptung, dass ℕ aus der unendlichen Mengenvereinigung
{1} ∪ {2} ∪ {3} ∪ ... = ℕ resultiert, führt zu folgender Konsequenz
Das tut sie natürlich keineswegs. Jedenfalls ist das ein Thema der Philosophie und nicht unumstritten. Siehe: https://en.wikipedia.org/wiki/Supertask

Aber nehmen wir "for the sake of the argument" einmal an, dass es so wäre, wie Du sagst. Was würde also daraus "folgen"?
Post by Ganzhinterseher
Geben wir die Zahl 1 in ein Reservoir, sodann die Zahl 2, sodann die Zahl 3 und fahren unbegrenzt so fort, indem wir z. B. die Geisterstunde nutzend um 0:00 Uhr beginnen, um 0:30 Uhr den zweiten Schritt ausführen, um 0:45 Uhr den
dritten usw., dann enthält das Reservoir nach unendlich vielen Schritten, Schlag 1:00 Uhr, den Grenzwert der Mengenfolge, die Menge aller natürlichen Zahlen.
Aber diese Behauptung hat auch eine andere Konsequenz: Geben wir um 0:00 Uhr die Zahlen 1 bis 10 in das Reservoir und entnehmen sogleich wieder die Zahl 1, um 0:30 Uhr geben wir die Zahlen 11 bis 20 in das Reservoir und entnehmen sogleich wieder die Zahl 2, um 0:45 Uhr geben wir die Zahlen 21 bis 30 in das Reservoir und entnehmen sogleich wieder die Zahl 3 usw., dann enthält das Reservoir nach unendlich vielen Schritten um 1:00 Uhr nichts, denn von jeder natürlichen Zahl kann der Zeitpunkt angegeben werden, zu dem sie wieder entnommen wurde.
Das Beispiel lässt sich sehr vereinfachen. (Auch wenn Dir das nicht gefallen wird, weil damit der ANSCHEIN verschwindet, es hier mit einem "Paradoxon" zu tun zu haben.)

Wir starten in beiden Fällen mit einem Reservoir, das schon alle natürlichen Zahlen enthält. :-)

Fall 1: Wir tun bei jedem Schritt genau GAR NICHTS. Ergebnis: "...dann enthält das Reservoir nach unendlich vielen Schritten, Schlag 1:00 Uhr [weiterhin] alle natürlichen Zahlen."

Fall 2: "um 00:00 entnehmen wir ... die Zahl 1, um 0:30 Uhr ... entnehmen [wir] die Zahl 2, um 0:45 Uhr ... entnehmen [wir] die Zahl 3 usw., dann enthält das Reservoir nach unendlich vielen Schritten um 1:00 Uhr nichts, denn von jeder natürlichen Zahl kann der Zeitpunkt angegeben werden, zu dem sie [...]" entnommen wurde."

So ist es. Jedenfalls kann man den "Supertask" mathematisch so beschreiben, dass dies der "zu erwartende" Ausgang des "Supertasks" ist (es sind dazu allerdings noch ein paar Zusatzannahmen erforderlich, auf die ich hier nicht näher eingehen möchte).

Wo siehst Du hier ein Problem?
Post by Ganzhinterseher
Die letztere Überlegung ist offensichtlich falsch
Huh?! "Offensichtlich?"

Sie kann nicht "offensichtlich falsch" sein, wenn sie allem Anschein nach richtig ist, Mückenheim. :-)
Post by Ganzhinterseher
denn <brabbel>
Ne, da ist nix.
Post by Ganzhinterseher
Und was noch deutlicher den Fehler aufzeigt,
Oha, NOCH deutlicher als GAR NICHT. Jetzt bin ich aber gespannt!
Post by Ganzhinterseher
ist dies: Würden wir statt der Zahlen 1, 2, 3, ... die Zahlen 10, 20, 30, ...
entnehmen, so enthielte das Reservoir um 1:00 Uhr [weiterhin] unendlich viele
Zahlen, nämlich alle nicht durch 10 teilbaren natürlichen Zahlen.
Äh, in der Tat...

Das ist offensichtlich RICHTIG!

"Entnommen" hat man am Ende IN DIESEM FALL also die Zahlen

10, 20, 30, ...

und weiterhin enthalten sind am Ende IN DIESEM FALL die Zahlen:

1, 2, 3, 4, 5, 6, 7, 8, 9, 11, 12, 13, ...

Was ist daran nun ungewöhnlich?

Im Fall 2 haben wir IM GEGENSATZ DAZU ja ALLE Zahlen (nacheinander) entnommen. Dann bleibt NATÜRLICH am Ende auch keine mehr übrig. Findest Du das verwunderlich? :-)
Post by Ganzhinterseher
Das Ergebnis eines Prozesses darf aber nicht von Bezeichnungswillkür abhängen.
Schwafel nicht so einen Unsinn daher, Mückenheim. Es ist ein "sachlicher" Unterschied, ob ich Dir 10 Kugeln vorlege und du NACHEINANDER eine nach der anderen wegnimmst (so dass am Ende KEINE mehr übrig bleibt), oder ob Du lediglich die zehnte Kugel wegnimmst. Mit "Bezeichnungswillkür" hat das nichts zu tun. *stöhn*
Post by Ganzhinterseher
Das <bla bla>
Wir fassen zusammen: Dein Beispiel hat NICHTS GEZEIGT, was in irgend einer Weise geeignet wäre, die "Mengenlehre" in Frage zu stellen. :-)

Das wirft natürlich auch ein schräges Licht auf Zeilbergers Äußerung: "[...] the bottom of page 112 and the top of page 113 [...] prove, once and for all, that (at least) the actual infinity is pure nonsense."

*lol*
Ganzhinterseher
2020-03-30 19:19:41 UTC
Permalink
Post by Me
Setzen wir hier einmal voraus, dass es sinnvoll ist, sogenannte Supertasks zu betrachten. (Hinweis: Die Mengenlehre setzt derartige "Supertasks" nicht voraus.
Das Abzählen der Brüche und die Cantor-Liste sind solche Supertasks.
Post by Me
Vielmehr erlaubt erst die ML sich auf einer rationalen Basis mit derartigen Supertasks zu beschäftigen.)
Wie Dagobert und Donald zeigen, aber nicht mit rationalem Ergebnis.
Post by Me
Post by Ganzhinterseher
Die letztere Überlegung ist offensichtlich falsch
Huh?! "Offensichtlich?"
Sie kann nicht "offensichtlich falsch" sein, wenn sie allem Anschein nach richtig ist,
Sie ist falsch, weil Dagobert und Donald alle natürlichen Zahlen erhalten (also dieselbe Menge) und beide auch gleichviele Zahlen zurückgeben. Man kann den Film genau synchron ablaufen lassen. Dann haben beide bis zu jedem Zeitpunkt vor 1 Uhr genau dieselbe Menge zurückgegeben. Um 1 Uhr ist aber Dagobert pleite, Donald reich. Diese Variante zu vertreten, ist einfach hirnlos. Und selbst wenn die Mengenlehre es erfordert, dann ist sie, wie eine schlechte Straßenlaterne, die lediglich ihren eigenen Pfahl beleuchtet, zu nichts anderem nütze als die Arbeitskraft intelligenter Menschen für sinnlose Tätigkeiten zu binden. Vermutlich hat die ML viel größeren wirtschaftlichen Schaden verursacht als der Corona Virus.

Gruß, WM
Me
2020-03-31 01:56:42 UTC
Permalink
Post by Me
Hinweis: Die Mengenlehre setzt derartige "Supertasks" nicht voraus.
Das Abzählen der Brüche und <bla bla> sind solche Supertasks.
Nein, "das Abzählen" der Brüche ist kein Supertask, weil das niemand außer Ihnen als Prozess auffasst.

Vielmehr gibt man dazu einfach eine BIJEKTION zwischen IN und der Menge der Brüche an. Das geht auch ganz OHNE "Supertask".

Siehe dazu: https://de.wikipedia.org/wiki/Cantorsche_Paarungsfunktion
Post by Me
Vielmehr erlaubt erst die ML sich auf einer rationalen Basis mit derartigen
Supertasks zu beschäftigen.
Siehe dazu: https://en.wikipedia.org/wiki/Supertask

(Man achte auf die Begriffe: "countably infinite", "uncountably infinite" und "ordinal number".)
Sie ist falsch, weil Dagobert und Donald alle natürlichen Zahlen erhalten (also dieselbe Menge) und beide auch gleichviele Zahlen zurückgeben.
Sie verwechseln hier "Anzahl" mit "Realität" (wie Cantor wohl gesagt hätte):

"Sei M die Gesamtheit (nü) aller endlichen Zahlen nü, M' die
Gesamtheit (2nü) aller geraden Zahlen 2nü. Hier ist unbedingt richtig, daß
M seiner Entität nach /reicher/ ist, als M'; enthält doch M außer den
geraden Zahlen, aus welchen M' besteht, noch außerdem alle ungeraden
Zahlen M''. Andererseits ist ebenso unbedingt richtig, daß den beiden
Mengen M und M' nach Nr. 2 und 3 /dieselbe/ Kardinalzahl zukommt. Beides
ist sicher und keines steht dem andern im Wege, wenn man nur auf die
Distinktion von /Realität/ und /Zahl/ achtet. Man muß also sagen: /die
Menge M hat mehr Realität wie M', weil sie M' und außerdem M'' als
Bestandteile enthält; die den beiden Mengen M und M' zukommenden
Kardinalzahlen sind aber gleich/. Wann endlich werden alle Denker
diese so einfachen und einleuchtenden Wahrheiten (gewiß nicht zu ihrem
Nachteile) anerkennen?" (Cantor)

Das wurde hier aber schon zig mal erwähnt und erklärt, so dass ich nicht noch einmal darauf eingehen möchte.
Me
2020-03-30 02:44:07 UTC
Permalink
Post by Roalto
Sein grosses Vorbild ist der Ultrafinitist Doron Zeilberger, dem hat er auch
seine Arbeiten präsentiert. Der meinte nur, er habe keine Lust das zu lesen,
da es sich nur um Philosophie handelt.
Das stellt sich mir anders dar. (Vielleicht beziehst Du Dich aber auf eine andere Quelle als die, die mir bekannt ist.)
Ganzhinterseher
2020-03-30 12:13:45 UTC
Permalink
Post by Me
Es ist *völlig unklar*, was das überhaupt sein soll. Der Begriff ist auch in der einschlägigen Literatur nicht dokumentiert. Kurz: Er macht keinen Sinn.
Es gibt das potentiell Unendliche in der Mathematik. Die Menge der ununterbrochenen Folgen der Ziffer 7 in der Dezimaldarstellung von pi (oder auch von allen irrationalen Zahlen) ist potentiell unendlich. Die Menge der Primzahllücken ist potentiell unendlich. Die Menge der Anfangsabschnitte der natürlichen Zahlen ist potentiell unendlich. Eigentlich gibt es überhaupt keine andere Unendlichkeit in der Mathematik.

Gruß, WM
Me
2020-03-30 13:26:15 UTC
Permalink
Post by Ganzhinterseher
Post by Me
Es ist *völlig unklar*, was das überhaupt sein soll. Der Begriff ist auch
in der einschlägigen Literatur nicht dokumentiert. Kurz: Er macht keinen
Sinn.
Es gibt das potentiell Unendliche in der Mathematik.
Hatte ich DAS bestritten? :-)

Hinweis: Nein.

Ich hatte geschrieben:

"WMs Gerede von "potentiell unendlichen Mengen" ist wirklich unerträglich.

Es ist *völlig unklar*, was das überhaupt sein soll. Der Begriff ist auch in der einschlägigen Literatur nicht dokumentiert. Kurz: Er macht keinen Sinn.

Scheißdreck halt."
Post by Ganzhinterseher
Die Menge der ununterbrochenen Folgen der Ziffer 7 in der Dezimaldarstellung
von pi (oder auch von allen irrationalen Zahlen) ist potentiell unendlich.
Die Menge der Primzahllücken ist potentiell unendlich. Die Menge der
Anfangsabschnitte der natürlichen Zahlen ist potentiell unendlich.
Wie ich schon sagte: Scheißdreck halt.
Post by Ganzhinterseher
Eigentlich gibt es überhaupt keine andere Unendlichkeit in der Mathematik.
Spätstens seit Cantor hat das sog. "aktual Unendliche" Eingang in die Mathematik gefunden. Im Kontext der sog. "klassischen Mathematik" ist das auch die einzige "Spielart" des Unendlichen, die von Relevanz ist. D. h. hier gibt es nur noch die Unterscheidung zwischen endlich und unendlich (=nicht endlich) - insbesondere in Bezug auf Mengen. :-)

Das "potentiell Unendliche" sielt in der heutigen Mathematik -wenn überhaupt- nur noch eine sehr untergeordnete Rolle.
Me
2020-03-30 13:37:51 UTC
Permalink
Post by Me
Spätstens seit Cantor hat das sog. "aktual Unendliche" Eingang in die
Mathematik gefunden. Im Kontext der sog. "klassischen Mathematik" ist das auch
die einzige "Spielart" des Unendlichen, die von Relevanz ist. D. h. hier gibt
es nur noch die Unterscheidung zwischen endlich und unendlich (=nicht endlich)
- insbesondere in Bezug auf Mengen. :-)
Das "potentiell Unendliche" spielt in der heutigen Mathematik -wenn überhaupt-
nur noch eine sehr untergeordnete Rolle.
"[...] potential infinity is almost forgotten now. In the ZFC set theory mindset, people tend not to even remember that distinction. They just think infinity means actual infinity and that's all there is to it." [S. Simpson quoted in N. Wolchover: "Dispute over infinity divides mathematicians",
Scientific American (3 Dec 2013)]

Im Kontext der ZFC ist das aber auch so. :-)
Helmut Richter
2020-03-30 14:28:40 UTC
Permalink
Post by Me
Spätstens seit Cantor hat das sog. "aktual Unendliche" Eingang in die
Mathematik gefunden. Im Kontext der sog. "klassischen Mathematik" ist
das auch die einzige "Spielart" des Unendlichen, die von Relevanz ist.
D. h. hier gibt es nur noch die Unterscheidung zwischen endlich und
unendlich (=nicht endlich) - insbesondere in Bezug auf Mengen. :-)
Ich halte nicht von vornherein für ausgeschlossen, dass es Zusammenhänge
geben mag, in denen eine solche Unterscheidung sinnvoll ist, die sich in
der Mainstream-Mathematik nicht durchgesetzt hat. Es hat ja schon öfter
Nicht-Mainstream-Logiken in der Mathematik gegeben, die sich nicht
durchgesetzt haben, ohne deswegen in sich falsch zu sein. Nur: die
Verfechter sollten dann auch klar sagen und definieren können, was sie
vorhaben und worin sich ihr Ansatz vom Mainstream unterscheidet. Und daran
fehlt es eben bei WM. Ich habe von ihm noch *nie* eine Definition gelesen
und auch noch nie einen Beweis, sondern nur Appelle an die Zustimmung der
Einsichtigen und Beschimpfungen der Uneinsichtigen. Das ist keine
Mathematik; deswegen lese ich das nicht mehr – mit wenigen Ausnahmen, wenn
mich *selbst* interessiert, wie man etwas auch exakt sagen könnte (z.B.
Limes von Folgen von Zahlenfolgen, was durchaus geht), aber dafür
interessiert sich niemand, auch WM selbst nicht.
Post by Me
Das "potentiell Unendliche" sielt in der heutigen Mathematik -wenn
überhaupt- nur noch eine sehr untergeordnete Rolle.
Manchmal habe ich den Verdacht, es stünde manchmal – nicht immer! – für
endlich aber unbeschränkt, etwa: die Mächtigkeit einer endlichen Mange ist
potentiell unendlich, d.h. nicht durch eine natürliche Zahl beschränkt.
--
Helmut Richter
Me
2020-03-30 14:52:31 UTC
Permalink
Post by Helmut Richter
Manchmal habe ich den Verdacht, es stünde manchmal – nicht immer! – für
endlich aber unbeschränkt
Eine Sichtweise, die in Bezug auf "Prozesse" vielleicht sinnvoll sein mag. (Sag ich jetzt mal so.)
Post by Helmut Richter
etwa: die Mächtigkeit einer endlichen Mange ist potentiell unendlich,
???

Wenn eine Menge endlich ist, dann ist sie endlich. Eine (bestimmte) Menge kann auch nicht wachsen oder kleiner werden, etc.
Post by Helmut Richter
d.h. nicht durch eine natürliche Zahl beschränkt.
Falls die von Dir betrachtete Menge c IN ist, würde ich sie dann (in diesem Fall) für _unendlich_ halten. Du nicht?

Vielleicht meinst Du aber eher eine "Kollektion" von Mengen:

{{}, {0}, {0,1}, {0,1,2}, ...} ?

Aber h i e r sind dann halt sämtliche Mengen der "Kollektion" endlich und die "Kollektion" (die im Kontext der ML ja selbst eine Menge ist) wiederum _unendlich_. Wo da der Begriff "potentiell unendlich" -sinvoll- reinkommen soll, sehe ich nicht.

Vielleicht kannst Du Deine Idee/Deinen Gedanken näher erklären?

(Hinweis: Ich fürchte in einem "klassischen Rahmen" macht die Sprechweise von "potentiell unendlich" generell keinen Sinn. Ich lasse mich aber gerne eines Besseren belehren. Am ehesten kann ich den Begriff noch im Hinblick auf das Band einer TM nachvollziehen: endlich aber -bei Bedarf- beliebig verlängerbar. Aber selbst hier kann man "Einwände" dagegen erheben...)
Michael Klemm
2020-03-30 15:07:29 UTC
Permalink
Post by Me
Post by Helmut Richter
Manchmal habe ich den Verdacht, es stünde manchmal – nicht immer! – für
endlich aber unbeschränkt
Eine Sichtweise, die in Bezug auf "Prozesse" vielleicht sinnvoll sein mag. (Sag ich jetzt mal so.)
Post by Helmut Richter
etwa: die Mächtigkeit einer endlichen Mange ist potentiell unendlich,
???
Vielleicht aber: "Die Mächtigkeit endlicher Mengen ist potentiell unendlich (bzw. unbegrenzt)".
Post by Me
Wenn eine Menge endlich ist, dann ist sie endlich. Eine (bestimmte) Menge kann auch nicht wachsen oder kleiner werden, etc.
Post by Helmut Richter
d.h. nicht durch eine natürliche Zahl beschränkt.
Falls die von Dir betrachtete Menge c IN ist, würde ich sie dann (in diesem Fall) für _unendlich_ halten. Du nicht?
{{}, {0}, {0,1}, {0,1,2}, ...} ?
Aber h i e r sind dann halt sämtliche Mengen der "Kollektion" endlich und die "Kollektion" (die im Kontext der ML ja selbst eine Menge ist) wiederum _unendlich_. Wo da der Begriff "potentiell unendlich" -sinvoll- reinkommen soll, sehe ich nicht.
Vielleicht kannst Du Deine Idee/Deinen Gedanken näher erklären?
(Hinweis: Ich fürchte in einem "klassischen Rahmen" macht die Sprechweise von "potentiell unendlich" generell keinen Sinn. Ich lasse mich aber gerne eines Besseren belehren. Am ehesten kann ich den Begriff noch im Hinblick auf das Band einer TM nachvollziehen: endlich aber -bei Bedarf- beliebig verlängerbar. Aber selbst hier kann man "Einwände" dagegen erheben...)
Me
2020-03-30 15:44:22 UTC
Permalink
Post by Michael Klemm
Vielleicht aber: "Die Mächtigkeit endlicher Mengen ist potentiell unendlich (bzw. unbegrenzt)".
Ja, das gefällt mir schon etwas besser. :-P

Zumal ja in ZFC die "Gesamtheit" aller "Mächtigkeiten" (Kardinalzahlen) nicht als Menge existiert. Daher kann man (in diesem Kontext) auch nicht sagen: "Die Menge aller Kardinalzahlen ist unendlich."
Ganzhinterseher
2020-03-30 16:06:41 UTC
Permalink
Post by Michael Klemm
Post by Me
Post by Helmut Richter
Manchmal habe ich den Verdacht, es stünde manchmal – nicht immer! – für
endlich aber unbeschränkt
Eine Sichtweise, die in Bezug auf "Prozesse" vielleicht sinnvoll sein mag. (Sag ich jetzt mal so.)
Post by Helmut Richter
etwa: die Mächtigkeit einer endlichen Mange ist potentiell unendlich,
???
Vielleicht aber: "Die Mächtigkeit endlicher Mengen ist potentiell unendlich (bzw. unbegrenzt)".
Richtig. Das ist eines von vielen Beispiele. Die Mächtigkeit absteigender Ordinalzahlfolgen oder die Mächtigkeit von Primzahllücken, oder die Größen von Kreisradien. Es gibt potentiell unendlich viele Beispiele.

Gruß, WM
Me
2020-03-30 17:41:16 UTC
Permalink
Post by Michael Klemm
Vielleicht aber: "Die Mächtigkeit endlicher Mengen ist potentiell unendlich (bzw. unbegrenzt)".
Opps... Hab's jetzt nocheinmal etwas genauer gelesen.

Ne, sorry, macht m. E. auch keinen Sinn. Abgesehen davon, dass das keine "Standardterminologie" ist.

Abgesehen davon ist die Mächtigkeit endlicher Mengen sehr wohl begrenzt, denn es gilt für alle Mengen M:

endlich(M) -> card(M) < aleph_0 .

Dass die Menge der Kardinalzahlen "der endlichen Mengen" nicht durch eine natürliche Zahl beschränkt sein kann (nach oben), sollte klar sein. :-P

Denn card({0, 1, ..., n}) = n+1 für ALLE n e IN; und {0, 1, ..., n} ist endlich für ALLE n e IN.

Auch wenn das für Mückenheim schwer vorzustellen ist: "Die Menge der endlichen Kardinalzahlen ist unendlich" - daran ist nichts "potentiell".
Helmut Richter
2020-03-30 16:43:14 UTC
Permalink
Post by Me
Post by Helmut Richter
Manchmal habe ich den Verdacht, es stünde manchmal – nicht immer! – für
endlich aber unbeschränkt
Eine Sichtweise, die in Bezug auf "Prozesse" vielleicht sinnvoll sein mag. (Sag ich jetzt mal so.)
Post by Helmut Richter
etwa: die Mächtigkeit einer endlichen Mange ist potentiell unendlich,
???
{{}, {0}, {0,1}, {0,1,2}, ...} ?
„eine“ steht im mathematischen Jargon oft für „jede“. Die Definition „eine
Gruppe heißt abelsch, wenn für sie gilt: ...“ soll bedeuten „jede Gruppe
heißt abelsch, für die gilt: ...“. Die Definition gilt ja nicht nur für
eine einzige Gruppe. So wars gemeint.

Das Beispiel war vermutlich zu primitiv, um irgendetwas auszusagen. Es
gibt aber nichttriviale Aussagen, die für beliebig große endliche Mengen,
nicht aber für unendliche gelten. Spontan fallen einem da natürlich
Aussagen aus der Mengenlehre selbst ein, wie „lässt sich nicht bijektiv
auf eine echte Teilmenge abbilden“, aber solche meine ich nicht.

Beispiel 1: Mit einer rekursiven Definition (zum Beispiel arithmetische
Ausdrücke in einer Programmiersprache) werden beliebig lange, beliebig
tief verschachtelte Ausdrücke definiert, aber keine unendlich langen oder
unendlich tief verschachtelten. Es gibt genug Menschen, auch sonst recht
gebildete, die sich schwer tun, das auseinanderzuhalten.

Beispiel 2 (mein Lieblingsbeispiel dazu): Für jede natürliche Zahl n
lassen sich n Punkte in der Ebene angeben, so dass je zwei von ihnen
ganzzahligen Abstand haben und dabei keine drei auf einer Geraden liegen.
Es lassen sich aber nicht unendlich viele Punkte mit dieser Eigenschaft
angeben.
--
Helmut Richter
Me
2020-03-30 17:28:31 UTC
Permalink
Post by Helmut Richter
Post by Me
Post by Helmut Richter
etwa: die Mächtigkeit einer endlichen Mange ist potentiell unendlich,
???
{{}, {0}, {0,1}, {0,1,2}, ...} ?
„eine“ steht im mathematischen Jargon oft für „jede“.
Das ist mir schon klar. :-P
Post by Helmut Richter
Die Definition „eine Gruppe heißt abelsch, wenn für sie gilt: ...“ soll
bedeuten „jede Gruppe heißt abelsch, für die gilt: ...“. Die Definition
gilt ja nicht nur für eine einzige Gruppe.
Auch das ist hinlänglich bekannt. (Heißt: Man ist damit vertraut.)
Post by Helmut Richter
So wars gemeint.
Dann verstehe ich es NOCH WENIGER. :-P

Wir brauchen das aber nicht zu vertiefen: Ich gehe davon aus, dass Du so etwas "in der Praxis" auch nie sagen würdest (bzw. gesagt hast). :-P
Post by Helmut Richter
Das Beispiel war vermutlich zu primitiv, um irgendetwas auszusagen. Es
gibt aber nichttriviale Aussagen, die für beliebig große endliche Mengen,
nicht aber für unendliche gelten.
Ja.
Post by Helmut Richter
Beispiel 1: Mit einer rekursiven Definition (zum Beispiel arithmetische
Ausdrücke in einer Programmiersprache) werden beliebig lange, beliebig
tief verschachtelte Ausdrücke definiert, aber keine unendlich langen oder
unendlich tief verschachtelten. Es gibt genug Menschen, auch sonst recht
gebildete, die sich schwer tun, das auseinanderzuhalten.
Vermutlich trifft das auch auf Herrn Mückenheim zu. :-P

Er verwendet gerne die "Schlussregel", dass etwas das für alle Mengen "der Form" {1, ..., n} (mit n e IN) gilt, auch für IN selbst gelten müsse. Ganz offensichtlich ist er sich der Irrigkeit dieses "Fehlschlusses" nicht bewusst.
Post by Helmut Richter
Beispiel 2 (mein Lieblingsbeispiel dazu): Für jede natürliche Zahl n
lassen sich n Punkte in der Ebene angeben, so dass je zwei von ihnen
ganzzahligen Abstand haben und dabei keine drei auf einer Geraden liegen.
Es lassen sich aber nicht unendlich viele Punkte mit dieser Eigenschaft
angeben.
Gab's da nicht einen schönen Aufsatz von Erdös dazu? Hab mir das mal angesehen. Nett! :-P (Den ersten Teil habe ich -so weit- sogar verstanden.)
Helmut Richter
2020-03-30 18:49:50 UTC
Permalink
Post by Me
Post by Helmut Richter
Beispiel 2 (mein Lieblingsbeispiel dazu): Für jede natürliche Zahl n
lassen sich n Punkte in der Ebene angeben, so dass je zwei von ihnen
ganzzahligen Abstand haben und dabei keine drei auf einer Geraden liegen.
Es lassen sich aber nicht unendlich viele Punkte mit dieser Eigenschaft
angeben.
Gab's da nicht einen schönen Aufsatz von Erdös dazu? Hab mir das mal
angesehen. Nett! :-P (Den ersten Teil habe ich -so weit- sogar
verstanden.)
Es ist nicht auf meinem Mist gewachsen. Ob ich es mittelbar oder unmittelbar
von Erdős habe, weiß ich nicht. Ich glaube, ich habe beide Richtungen
schließlich herausbekommen.

Man beweist es für Mengen mit rationalen Abständen, und wenn man eine Lösung
hat, bläst man sie mit dem Hauptnenner auf, um eine mit ganzzahligen Abständen
zu bekommen. So sieht man, dass der unendliche Fall nicht so geht wie der
endliche: dort gibt es keinen Hauptnenner. Die unendlich vielen Punkte mit
rationalen Abständen gibts auch im unendlichen Fall.

Wenn ich mich recht erinnere, gings so (nur die Beweisidee): Man nimmt
z.B. einen Kreis und darauf zwei Punkte, so dass sin und cos des halben
Öffnungswinkels vom Ursprung aus gesehen rational sind, also etwa den Punkt
<1, 0> und den Punkt <cos φ, sin φ> für geeignetes φ. Dann gilt der rationale
Abstand auch für <cos k·φ, sin k·φ> nach den Additionstheoremen. Zu zeigen
ist nur noch, dass sich der Kreis nicht schließt. Wenn man das nicht leicht
zeigen kann, nimmt man einfach ein so kleines φ, dass n·φ < 2π.
Pythagoräische Dreiecke gibts ja beliebig schlank.

Für die umgekehrte Richtung zeigt man die Verschärfung: Unendlich viele Punkte
mit ganzzahligen Abständen liegen notwendig *alle* auf einer Geraden. Enthält
die Menge nämlich ein Dreieck, gibt es nur endlich viele Punkte, die zu diesen
dreien ganzzahligen Abstand haben – sie liegen auf Hyperbelscharen mit
endlich vielen Schnittpunkten.

--
Helmut Richter
Ganzhinterseher
2020-03-30 15:59:54 UTC
Permalink
Post by Helmut Richter
Und daran
fehlt es eben bei WM. Ich habe von ihm noch *nie* eine Definition gelesen
Da hast Du wohl die folgende übersehen:

=============================================

de.sci.mathematik ›
Ist die Mathematik eine "Magd der Physik"?
34 Einträge von 12 Autoren
ich (Ganzhinterseher Ändern)
11:00 (vor 1 Stunde)
Am Dienstag, 3. März 2020 21:50:22 UTC+1 schrieb Helmut Richter:

Definition: E(k) = {k, k+1, k+2, ...} heißt Endsegmente der natürlichen Zahlen.
Definition ∩{E(k) | k ∈ ℕ } = { } heißt der Schnitt über alle Endsegmente.

Für alle Endsegmente E(k) ist der Schnitt

∩{E(1), E(2), ..., E(k)} = E(k)

und die Kardinalzahl |E(k)| = ℵo .
===========================================
Post by Helmut Richter
und auch noch nie einen Beweis
===========================================
Damit ist bewiesen, dass diese Endsegmente im Schnitt über alle Endsegmente entfallen können, ohne das Ergebnis zu ändern. Das erste Endsegment E(x), das nicht entfallen kann

∩{E(x), E(x+1), E(x+2), ...} = { }

ist keines der Endsegmente, für die gilt:

∀k ∈ ℕ: ∩{E(1), E(2), ..., E(k)} = E(k) /\ |E(k)| = ℵo .

================================================
Post by Helmut Richter
, sondern nur Appelle an die Zustimmung der
Einsichtigen und Beschimpfungen der Uneinsichtigen.
Hast Du meine direkte Erwiderung im Beitrag vom 4. März nicht gelesen? Oder hast Du die Definitionen nicht verstanden? Oder macht es Dir einfach nur Spaß, zu lügen und zu verleumden?

Gruß, WM
Me
2020-03-30 16:33:42 UTC
Permalink
Post by Ganzhinterseher
Post by Helmut Richter
Ich habe von ihm noch *nie* eine Definition gelesen
Jedenfalls keine korrekte.
Post by Ganzhinterseher
Definition: E(k) = {k, k+1, k+2, ...} heißt Endsegmente der natürlichen Zahlen.
*seufz* Mückenheim, irgendein Unsinn, dem man das Wort "Definition:" voranstellt, ist noch lange keine korrekte Definition (im mathematisch-logischen Sinne).

Allerdings muss ich zugeben, dass Du hier schon vergleichsweise nahe an einer korrekten Definition dran warst.

Besser aber so: Für k e IN heißt E(k) = {k, k+1, k+2, ...} /Endsegment_zu_k/. Z. B. (Streng formal wäre "E(k) = {n e IN: n >= k}" hier noch besser.)

Danach kann man definieren:

Endsegment(E) :<-> En e IN: E = E(n) .

E ist also ein /Endsegment/ genau dann, wenn es eine natürliche Zahl n gibt, so dass E ein Endsegment_zu_k ist.

Damit wäre dann

{E(n) : n e IN}

die Menge aller Endsegmente. :-)

Denn es gilt:

E e {E(n) : n e IN} <-> En e IN. E = E(n) ,

mit anderen Worten:

E e {E(n) : n e IN} <-> Endsegment(E) ,

für bel. Mengen E.
Post by Ganzhinterseher
Definition ∩{E(k) | k ∈ ℕ} = { } heißt der Schnitt über alle Endsegmente.
Naja... Das muss man eigentlich nicht "definieren". Wenn {E(k) | k ∈ ℕ} die Menge aller Endsegmente ist, und das ist sie (siehe oben), dann kann man (nach einer üblichen Konvention) ∩{E(k) | k ∈ ℕ} den Schnitt "über alle Endsegmente" nennen.

Viel gewonnen wird durch solche simplen SPRACHLICHEN Festlegungen/Konventionen allerdings nicht.

Das versteht natürlich jemand, der Mathematik mit "Sprachmagie" verwechselt, nicht.
Post by Ganzhinterseher
Für alle Endsegmente E(k) ist der Schnitt
∩{E(1), E(2), ..., E(k)} = E(k)
und die Kardinalzahl |E(k)| = ℵo .
Das ist wieder mal Geschwurbel.
Post by Ganzhinterseher
Für alle k e IN ist der Schnitt
∩{E(1), E(2), ..., E(k)} = E(k)
und die Kardinalzahl |E(k)| = ℵo .
Bzw. streng formal:

Ak e IN: ∩{E(1), E(2), ..., E(k)} = E(k) & |E(k)| = ℵo

oder

Ak e IN: ∩{E(1), E(2), ..., E(k)} = E(k)

und

Ak e IN: |E(k)| = ℵo .

Das ist aber lediglich eine mathematische Aussage/Behauptung. Also WEDER eine Definition, NOCH ein Beweis (für irgendetwas).
Post by Ganzhinterseher
Post by Helmut Richter
und auch noch nie einen Beweis
Ja, geht mir auch so.
Post by Ganzhinterseher
Damit ist bewiesen, <bla>
Gar nichts ist damit "bewiesen". Oder genauer: Obige Aussage/Behauptung IST KEIN BEWEIS. Vielmegr ist SIE SELBST eines Beweises bedürftig, Sie <grumpf>.
Post by Ganzhinterseher
dass <ab hier kommt nur noch Unsinn>.
Post by Helmut Richter
sondern nur Appelle an die Zustimmung der Einsichtigen
und Beschimpfungen der Uneinsichtigen.
Jep. So ist es.
Ganzhinterseher
2020-03-30 17:41:16 UTC
Permalink
Post by Me
Ak e IN: |E(k)| = ℵo .
Das ist aber lediglich eine mathematische Aussage/Behauptung. Also WEDER eine Definition, NOCH ein Beweis (für irgendetwas).
Es ist ein Beweis dafür, dass jedes dieser Endsegmente aus

∩{E(k) | k ∈ ℕ } = { }

fortfallen kann, ohne das Ergebnis zu ändern.
Post by Me
Gar nichts ist damit "bewiesen".
Dann beweise doch das Gegenteil, indem Du ein Endsegment definierst, das nicht fortfallen kann.

Auch in der Mengenlehre gilt: Wenn kein Element einer Menge existiert, dann ist diese Menge leer. Und jede wohlgeordnete Menge muss ohnehin ein niederstes Element haben, um zu existieren.
Post by Me
Oder genauer: Obige Aussage/Behauptung IST KEIN BEWEIS.
Obige Aussage ist eine Beweis dafür, dass Matheologie keine Logischen Schlüsse erlaubt. Aber das ist ja nichts Neues.

Wenn Dagobert Duck seine Operation durchführt und Donald Duck dasselbe tut, parallel zu Dagobert, aber nicht die jeweils kleinste Zahl n zurückgibt, sondern die Zahl 10n, dann haben beide Punkt 1 Uhr genau dieselbe Anzahl von Transaktionen vorgenommen. Trotzdem ist Dagobert pleite und Donald reich.

Gruß, WM
Me
2020-03-30 17:56:05 UTC
Permalink
dass jedes dieser ...
Du bist einfach zu blöde dafür, den Unterschied zwischen einer Aussage, die sich auf "jedes einzelne" Element eines "Kollektivs" bezieht und einer Aussage, die sich auf "alle Elemente (des Kollektivs) zusammen" bezieht, zu begreifen. So viel Blödheit ist selten!

Sei A = {1, 2, 3}.

Ja, für jedes x e A gilt: A \ {x} =/= {}.

Nein, A \ {1, 2, 3} ist natürlich NICHT =/= {}, sondern = {} .
Wenn Dagobert Duck seine Operation durchführt und Donald Duck dasselbe tut,
parallel zu Dagobert, aber nicht die jeweils kleinste Zahl n zurückgibt,
sondern die Zahl 10n, dann haben beide Punkt 1 Uhr genau dieselbe Anzahl
von Transaktionen vorgenommen. Trotzdem ist Dagobert pleite und Donald reich.
aleph_0 = 10 * aleph_0 .

Hinweis: Wenn ich die aleph_0 Elemente {1, 2, 3, ...} von IN subtrahiere, ist die resultierende Menge leer. Wenn ich aber die aleph_0 Elemente {2, 4, 3, ...} subtrahiere, dann bleiben aleph_0 Elemente übrig. Offenbar ein mengentheoretischer Sachverhalt, den Du noch nie kapiert hast (wie dein obiges "Beispiel" zeigt).
Ganzhinterseher
2020-03-30 18:58:36 UTC
Permalink
Post by Me
Unterschied zwischen einer Aussage, die sich auf "jedes einzelne" Element eines "Kollektivs" bezieht und einer Aussage, die sich auf "alle Elemente (des Kollektivs) zusammen" bezieht
Alle definierbaren Endsegmente

∀k ∈ ℕ: ∩{E(1), E(2), ..., E(k)} = E(k) /\ |E(k)| = ℵo

können aus

∩{E(k) | k ∈ ℕ } = { }

entfernt werden. Eine Aussage, die den Allquantor beinhaltet gilt für die gesamte Menge. Ein Kollektive, das keine Elemente hat, ist leer.
Post by Me
Sei A = {1, 2, 3}.
Ja, für jedes x e A gilt: A \ {x} =/= {}.
Nein, A \ {1, 2, 3} ist natürlich NICHT =/= {}, sondern = {} .
Jedes x einzeln, aber nicht alle drei zugleich. In meinem Beispiel kann man dagegen alle definierbaren Endsegmente zugleich herausnehmen, ohne den leeren Schnitt zu verändern. Beweis: Du kannst kein Endsegment definieren, das benötigt wird.
Post by Me
Post by Ganzhinterseher
Wenn Dagobert Duck seine Operation durchführt und Donald Duck dasselbe tut,
parallel zu Dagobert, aber nicht die jeweils kleinste Zahl n zurückgibt,
sondern die Zahl 10n, dann haben beide Punkt 1 Uhr genau dieselbe Anzahl
von Transaktionen vorgenommen. Trotzdem ist Dagobert pleite und Donald reich.
aleph_0 = 10 * aleph_0 .
Das ist kein Beweis für die Richtigkeit der ML, sondern ein Beweis für das mangelnde Denkvermögen ihrer Anhänger. Die identische Anzahl von zurückgegeben Dollars verbietet ein unterschiedliches Ergebnis.
Post by Me
Hinweis: Wenn ich die aleph_0 Elemente {1, 2, 3, ...} von IN subtrahiere, ist die resultierende Menge leer.
Interessant. Wenn man die gleiche Menge zu einer Umordnung von
0, 1, 2, 3, ...
verwendet, dann wird die hinter der Null stehende Menge

1, 2, 3, ..., n, 0, n+1, ...

nicht leer.
Post by Me
Wenn ich aber die aleph_0 Elemente {2, 4, 3, ...} subtrahiere, dann bleiben aleph_0 Elemente übrig. Offenbar ein mengentheoretischer Sachverhalt,
der jede Anwendung auf wissenschaftliche Probleme verbietet.

Gruß, WM
Me
2020-03-30 19:19:35 UTC
Permalink
In meinem Beispiel kann man dagegen alle [...] Endsegmente zugleich
herausnehmen, ohne den leeren Schnitt zu verändern.
Nö, weil außerhalb der Mückenheimschen Wahnwelt

SCHNITT{}

nicht notwendigerweise = {} ist. In NBG ist SCHNITT{} z. B gleich der Klasse aller Mengen, also =/= {}.

Hinweis: In ZFC ist "SCHNITT{}" vielfach noch nicht einmal definiert.
Die identische Anzahl von zurückgegeben Dollars verbietet ein unterschiedliches Ergebnis.
Nun ich weiß nicht, auf welcher Basis Du Deine Aussage tätigst, jedenfalls auf keiner mengentheoretischen.

Hinweis: Zum einen gilt card({1, 2, 3, ...}) = card({10, 20, 30, ...}),
zum anderen

IN \ {1, 2, 3, ...} = {}
und
IN \ {10, 20, 30, ...} = {1, 2, 3, 4, 5, 6, 7, 8, 9, 11, 12, 13, ...}

Du bist einfach zu blöde für Mathematik.
Ganzhinterseher
2020-03-30 19:37:21 UTC
Permalink
Post by Me
In meinem Beispiel kann man dagegen alle [...] Endsegmente zugleich
herausnehmen, ohne den leeren Schnitt zu verändern.
Nö, weil
SCHNITT{}
nicht notwendigerweise = {} ist.
Es ist aber unmöglich ein Endsegment zu definieren, das im Schnitt bleiben muss. Deswegen ist die Menge der definierbaren Endsegmente leer.

Das lässt sich per Induktion zeigen. Aber es ist auch hier schon klar

∀k ∈ ℕ: ∩{E(1), E(2), ..., E(k)} = E(k) /\ |E(k)| = ℵo.

Was Du auch versuchen magst: Jedes definierbare Endsegment kegele ich Dir raus, ohne dafür andere einzusetzen. Diskussion wird übrigens auch hier angewandt:

We go through the list and {{for}} every statement which begins with 'there exists' and is true in the universe, we pick out one set in the universe which makes it true. Since there are only countably many statements, we have chosen only countably many elements and we place them in M. Next we form all statements using these sets and again only look at those which begin with 'there exists'. If they are true in the universe, we pick out one set which makes them true and adjoin these to M. We repeat this process countably many times. The resulting collection of all sets so chosen is clearly countable. Now it is easy to see that the true statements of M are exactly the true statements in the universe. This is proved by induction on the number of quantifiers appearing at the beginning of the statement. If there are none, then the statement simply is composed of finitely many statements of the form 'x is a member of y', connected by the Boolean operators. This clearly is true in M if and only if it is true in the universe. Now consider a statement with one quantifier. By considering its negation, if necessary, we may assume it begins with 'there exists'. Now our choosing process clearly guarantees that the statement is true in M if and only if it is true in the universe. The proof now proceeds by a simple induction on the number of quantifiers.
You may feel that this argument is too simple to be correct, but I assure you that this is the entire argument, needing only a very simple argument to show that one can always bring the quantifiers to the front of the statement. I might add that the underlying reason the argument is so simple is because it applies to any system whatsoever, as long as we have only finitely many predicates (even countably many will work the same way) so that the number of statements that can be formed is countable. This theorem is perhaps a typical example of how a fundamental result which has such wide application must of necessity be simple." [Paul J. Cohen: "The discovery of forcing", Rocky Mountain Journal of Mathematics 32,4 (2002) p. 1076f]

Würde Dein Argument gelten, dann käme Skolem keine Gedeutung zu. Du bist ungefähr so primitiv fanatisch wie Zermelo.
Post by Me
Die identische Anzahl von zurückgegeben Dollars verbietet ein unterschiedliches Ergebnis.
Nun ich weiß nicht, auf welcher Basis Du Deine Aussage tätigst, jedenfalls auf keiner mengentheoretischen.
Mathematik. Wenn X Aktionen jeweils ein Element betreffen, dann sind X Elemente betroffen, gleichgültig wie sie heiße mögen.
Post by Me
Hinweis: Zum einen gilt card({1, 2, 3, ...}) = card({10, 20, 30, ...}),
Deswegen ist die ML Schrott.

Gruß, WM
Me
2020-03-30 20:35:41 UTC
Permalink
Würde Dein Argument gelten, dann <bla>
Ja, wie auch immer.

EOD
Ganzhinterseher
2020-03-30 20:51:43 UTC
Permalink
EOD
Nicht ganz: Wir wollen noch feststellen, dass mit jedem Endsegment, das in

E(1) ∩ E(2) ∩ E(3) ∩ ... = { }

ohne Änderung des Ergebnisses (oÄdE) fortfallen kann, auch alle Vorgänger fortfallen können. Das sollte selbst Dir klar sein. Also müsste es ein niederstes definierbares Endsegment geben, das nicht (oÄdE) fortfallen kann, wenn die Menge der definierbaren Endsegments, die nicht (oÄdE) fortfallen können, nicht leer ist. Darf man so viel schwere Logik bei Dir voraussetzen?

Das gibt es aber nicht. Die Mengenlehre verstößt somit gegen primitivste logische Zusammenhänge,

Gruß, WM
Me
2020-03-30 22:10:57 UTC
Permalink
Post by Ganzhinterseher
ohne Änderung des Ergebnisses (oÄdE) fortfallen kann
Ja, ja, es können sogar UNENDLICH viele Endsegmente "fortfallen" solange nur unendlich viele übrig bleiben.

- Ich glaube man hat Dir das schon ca. 1000 mal gesagt/erklärt.

Begriffen hast Du es offenbar immer noch nicht.

Da kann man dann aber auch wirklich nichts mehr machen.

Daher: EOD
Ganzhinterseher
2020-03-31 14:31:30 UTC
Permalink
Post by Me
Post by Ganzhinterseher
ohne Änderung des Ergebnisses (oÄdE) fortfallen kann
Ja, ja, es können sogar UNENDLICH viele Endsegmente "fortfallen" solange nur unendlich viele übrig bleiben.
Es können jedenfalls alle definierbaren Endsegmente fortfallen. Das lässt sich nämlich folgendermaßen beweisen:

∀k ∈ ℕ_def: ∩{E(1), E(2), ..., E(k)} = E(k) /\ |E(k)| = ℵo.

Das schwurbelige "solange unendlich viele übrigbleiben" enthält bewusst keinen Bezug zu definierbaren, denn man kann von allen beweisen, dass sie unnütz sind.
Post by Me
- Ich glaube man hat Dir das schon ca. 1000 mal gesagt/erklärt.
Begriffen hast Du es offenbar immer noch nicht.
Es ist ja auch falsch, denn die Menge der Endsegmente ist wohlgeordnet und bei Notwendigkeit von definierbaren, müsste eines das niederste notwendige definierbare sein. Es gibt aber keines.
Post by Me
Da kann man dann aber auch wirklich nichts mehr machen.
Man könnte beweisen, dass wenn E(n) nicht notwendig ist, auch E(n+1) nicht notwendig ist. Und wer behauptet, dass stattdessen ein Vorgänger von E(n+1) notwendig sein könnte, hat weder ZFC noch Denken gelernt.

Gruß, WM
Juergen Ilse
2020-03-31 15:32:37 UTC
Permalink
Hallo,
Post by Ganzhinterseher
Post by Me
Post by Ganzhinterseher
ohne Änderung des Ergebnisses (oÄdE) fortfallen kann
Ja, ja, es können sogar UNENDLICH viele Endsegmente "fortfallen" solange nur unendlich viele übrig bleiben.
∀k ∈ ℕ_def: ∩{E(1), E(2), ..., E(k)} = E(k) /\ |E(k)| = ℵo.
Das ist kein Beweis sondern "bluehender Bloedsinn".
Post by Ganzhinterseher
Das schwurbelige "solange unendlich viele übrigbleiben" enthält bewusst keinen Bezug zu definierbaren, denn man kann von allen beweisen, dass sie unnütz sind.
Das ist auffallend richtig, dass der Begriff "definierbare Endsegmente"
voellig unnuetz ist, da *alle* Endsegmente definierbar sind. "Solange
unedlich viele uebrig bleiben" ist natuerlich ungenau formuliert.
Man sollte besser sagen "solange die Restmenge unendlich ist".
Post by Ganzhinterseher
Post by Me
- Ich glaube man hat Dir das schon ca. 1000 mal gesagt/erklärt.
Begriffen hast Du es offenbar immer noch nicht.
Es ist ja auch falsch,
Nein, es ist beweisbar richtig und wurde hier bereits oft genug bewiesen.
Wenn der Schnitt unedlich vieler Endsegmente *nicht* leer waere, muesste
es mindestens eine natuerliche Zahl geben, die in *allen* von diesen unend-
lich vielen enthalten ist. Sei n (beliebig aber fest) eine solche Zahl.
Da die Menge der Endsegmente unendlich ist, gibt es in der "Indexmenge"
dieser Endsegmente keine groesste Zahl, sondern zu jeder beliebigen natuer-
lichen Zahl laesst sich in der Indexmenge mindestens eine finden, die
groesser als jene natuerliche Zahl ist. Insbesondere laesst sich in der
Indexmenge also eine natuerliche Zahl finden, die groesser als das oben
genannte n ist. Sei m eine solche Zahl aus der Indexmenge. Da m > n ist,
folgt daraus und aus der Definition der Endsegmente, dass das Endsegment
E(m) (das ja die Menge { k element IN | k >= m } ist) die Zahl n *nicht*
enthalten kann. Das ist ein Widerspruch zu der Voraussetzung, dass n im
Schnitt der unendlich vielen Endsegmente liegt, da wir eines dieser unend-
lich vielen Endsegmente gefunden haben, dass n *nicht* enthaelt.
q.e.d.
Post by Ganzhinterseher
denn die Menge der Endsegmente ist wohlgeordnet und bei Notwendigkeit
von definierbaren, müsste eines das niederste notwendige definierbare
sein. Es gibt aber keines.
Es gibt noch nicht einmal die "Menge der notwendigen Endsegmente", da das
keine Menge (und damit erst recht keine "wohlgeordnete Menge") ist. Diese
vermeintliche Menge ist nicht eindeutig bestimmt, und damit bestimmt keine
Menge (noch nicht einmal ein mathematisches Objekt sondern nur Bloedsinn).
Auch das wurde IHNEN bereits hinreichend oft erklaert. Ich hatte IHNEN
sogar *BEWIESEN*, dass es kein "minimales fuer den leeren Schnitt not-
wendiges Endsegment" geben kann.
Post by Ganzhinterseher
Man könnte beweisen, dass wenn E(n) nicht notwendig ist, auch E(n+1)
nicht notwendig ist.
SIE sind tatsaechlich mathematisch zu unfaehig, um Mengenlehre auch nur
im Ansatz zu verstehen.

Tschuess,
Juergen Ilse (***@usenet-verwaltung.de)

Me
2020-03-30 22:49:57 UTC
Permalink
Post by Ganzhinterseher
die ML Schrott.
Genau, Mückenheim, so wird es sein!
Juergen Ilse
2020-03-31 03:21:28 UTC
Permalink
Hallo,
Post by Ganzhinterseher
Post by Me
Unterschied zwischen einer Aussage, die sich auf "jedes einzelne" Element eines "Kollektivs" bezieht und einer Aussage, die sich auf "alle Elemente (des Kollektivs) zusammen" bezieht
Alle definierbaren Endsegmente
∀k ∈ ℕ: ∩{E(1), E(2), ..., E(k)} = E(k) /\ |E(k)| = ℵo
können aus
∩{E(k) | k ∈ ℕ } = { }
entfernt werden.
SIE haben versaeumt das zu beweisen (was kein Wunder ist, weil diese Aussage
falsch und damit eben *nicht* beweisbar iist).
Post by Ganzhinterseher
Eine Aussage, die den Allquantor beinhaltet gilt für die gesamte Menge.
Man kann in dem Fall nicht von Einzelelmenten auf eine Aussage ueber die
gesamte Menge schliessen. Beispiel gefaellig:

Wenn ich aus der Menge der natuerlichen Zahlen *eine* (beliebige) entferne,
bleibt die Restmenge unendlich (sprichMaechtigkeit aleph0). Das gilt fuer
*jede* natuerliche Zahl (nicht nur fuer "mueckenheim definierbare"). Folgt
daraus, dass ich aaus der Menge der natuerlichen Zahlen *alle* heraaus-
nehmen kann und die Restmenge noch immer die MAechtigkeit aleph0 hat?
Natuerlich nicht, denn wenn ich aus der Menge der natuerlichen Zahlen alle
natuerlichen Zahlen entnehme, dann ist die Restmenge die leere Menge, deren
Maechtigkeit ganz offensichtlich nicht aleph0 sondern 0 ist ...

Die Schllussweise "jede einzelne naatuerliche Zahl kann ich (einzelne) aus
der Menge der natuerlichen Zahlen entnehmen, ohne dass die Restmenge eine
andere Maechtigkeit als aleph0 haette, aalso kann ich aauch alle natuer-
lichen Zahlen aus der Menge der natuerlichen Zahlen heraausnehmen und die
(leere) Restmenge hat noch immer die Maechtigkeit aleph0" ist offensichtlich
kompletter Bloedsinn, es ist aber *exaakt* die Schlussweis, die SIE dort
oben anzuwenden verssuchen.

Tschuess,
Juergen Ilse (***@usenet-veerwaltung.de)
Juergen Ilse
2020-03-31 02:56:23 UTC
Permalink
Hallo,
Post by Ganzhinterseher
Post by Me
Ak e IN: |E(k)| = ℵo .
Das ist aber lediglich eine mathematische Aussage/Behauptung. Also WEDER eine Definition, NOCH ein Beweis (für irgendetwas).
Es ist ein Beweis dafür, dass jedes dieser Endsegmente aus
∩{E(k) | k ∈ ℕ } = { }
fortfallen kann, ohne das Ergebnis zu ändern.
Unfug.
Post by Ganzhinterseher
Post by Me
Gar nichts ist damit "bewiesen".
Stimmt.
Post by Ganzhinterseher
Dann beweise doch das Gegenteil, indem Du ein Endsegment definierst, das nicht fortfallen kann.
SIE haben noch nicht einmal verstanden, dass auch wenn *jedes* *einzelne*
Endsegment wegfallen kann ohne den Schnitt u aendern (solange nur das eine
wegfaellt) nicht das geringste darueber aussagt, ob der Wegfall *all* *dieser*
Endsegmente auf einmal ebenfalls keinen Einfluss auf den Schnitt haette (und
in der Tat waere eine solche Behauptung falsch).
Post by Ganzhinterseher
Auch in der Mengenlehre gilt: Wenn kein Element einer Menge existiert, dann ist diese Menge leer. Und jede wohlgeordnete Menge muss ohnehin ein niederstes Element haben, um zu existieren.
Die "Menge aller Endsegmente, die wegfaallen kann ohne dass der Schnitt
sich aendert" existiiert nicht, denn diese "Menge" ist nicht eindeutig
bestimmt und damit auch keine Menge (was IHNEN auch schon bewiesen wurde).
Post by Ganzhinterseher
Post by Me
Oder genauer: Obige Aussage/Behauptung IST KEIN BEWEIS.
Obige Aussage ist eine Beweis dafür,
... dass SIE voellig unfaehig zu jeglicher Art mathematischen denkens sind.
Ja, dem kann man durchaus zustimmen.

Tschuess,
Juergen Ilse (***@usenet-verwaltung.de)
Juergen Ilse
2020-03-30 18:05:43 UTC
Permalink
Hallo,
Post by Helmut Richter
Manchmal habe ich den Verdacht, es stünde manchmal – nicht immer! – für
endlich aber unbeschränkt, etwa: die Mächtigkeit einer endlichen Mange ist
potentiell unendlich, d.h. nicht durch eine natürliche Zahl beschränkt.
So stellt es sich WM vor, aber wie sollte das gehen? Wenn M eine rndliche
Menge und m die Anzahl ihrer Elemente ist, dann kann man immer mit m-1
Vergleichen ein groesstes Element der Menge ermitteln, dass dann natuerlich
auch obere Schranke dieser endlichen Menge ist. WM stellt sich das aber so
vor, dass dieser endlichen Menghe "bei Bedarf" eben immer neue Elemente
hinzugefuegt werden und man deshalb "die Menge der Elemente" nicht bestimmen
kann, obwohl die "Menge" zu jedem "betrachteten Zeitpunkt" dennoch nur end-
lich ist ... Das ist natuerlich (zumindest im Kontext von ZFC) kompletter
Nonsens ...

Tschuess,
Juergen Ilse (***@usenet-verwaltung.de)
Ganzhinterseher
2020-03-30 19:07:21 UTC
Permalink
Post by Juergen Ilse
Hallo,
Post by Helmut Richter
Manchmal habe ich den Verdacht, es stünde manchmal – nicht immer! – für
endlich aber unbeschränkt, etwa: die Mächtigkeit einer endlichen Mange ist
potentiell unendlich, d.h. nicht durch eine natürliche Zahl beschränkt.
So stellt es sich WM vor, aber wie sollte das gehen?
So wie die Primzahllücken?
So wie die Kreisradien?
So wie die Siebenerfolgen in Dezimaldarstellungen irrationaler Zahlen?
Post by Juergen Ilse
Wenn M eine rndliche
Menge und m die Anzahl ihrer Elemente ist, dann kann man immer mit m-1
Vergleichen ein groesstes Element der Menge ermitteln, dass dann natuerlich
auch obere Schranke dieser endlichen Menge ist. WM stellt sich das aber so
vor, dass dieser endlichen Menghe "bei Bedarf" eben immer neue Elemente
hinzugefuegt werden und man deshalb "die Menge der Elemente" nicht bestimmen
kann, obwohl die "Menge" zu jedem "betrachteten Zeitpunkt" dennoch nur end-
lich ist ... Das ist natuerlich (zumindest im Kontext von ZFC) kompletter
Nonsens ...
Deswegen ist ZFC im Kontext der Mathematik (die man auf wissenschaftliche Probleme anwenden kann) kompletter Nonsense.

Auch hier Dagobert und Donald: Beide erhalten jeweils 10 Dollars, Dagobert gibt den ersten zurück, Donald den letzten. Das wird unendlich oft durchgeführt, bis |N erschöpft ist. Jedenfalls führen beide exakt gleichviele Aktionen durch und geben also exakt gleichviele Dollars zurück. Trotzdem ist Dagobert am Ende pleite, Donald reich, im Gegensatz zu den "realen" Verhältnissen in Entenhausen. Kann wirklich jemand so beschränkt sein, das für gute Mathematik zu halten?

Gruß, WM
Hans Crauel
2020-03-30 20:30:53 UTC
Permalink
Helmut Richter schrieb
Post by Helmut Richter
Post by Me
Das "potentiell Unendliche" sielt in der heutigen Mathematik -wenn
überhaupt- nur noch eine sehr untergeordnete Rolle.
Manchmal habe ich den Verdacht, es stünde manchmal – nicht immer! –
für endlich aber unbeschränkt, etwa: die Mächtigkeit einer endlichen
Mange ist potentiell unendlich, d.h. nicht durch eine natürliche Zahl
beschränkt.
Ein bisschen in diese Richtung gehen die Schreib-/Sprechweisen
C^n für "n-mal stetig differenzierbar" und C^infty, was nicht
für "unendlich oft stetig diff'bar" steht, sondern (nur) für
"beliebig oft stetig diff'bar".

Hans
Ganzhinterseher
2020-03-30 15:07:20 UTC
Permalink
Post by Me
Aber WMs Gerede von "potentiell unendlichen Mengen" ist wirklich unerträglich.
Es ist *völlig unklar*, was das überhaupt sein soll.
Wir besprechen doch gerade ein sehr schönes Beispiel:

Durch Transpositionen kann die 1 aus

1, 2, 3, ...

an den Nachfolgern vorbeidefilieren:

2, 3, 4, ..., n-1, 1, n, n+1, n+2, ...

aber nicht an allen. Also muss ein Puffer vorhanden sein. Die Menge der Endsegmente, die diesen Puffer bildet, besitzt nach dem Wohlordnungssatz ein niederstes Element m. Aber man kann es nicht definieren, da 1 mit jeder definierbaren Zahl vertauscht werden kann.

Sollte es tatsächlich eine Überforderung sein, das zu verstehen?

Gruß, WM

Natürlich ist das m nicht definierbar, also eine dunkle Zahl. Denn an jeder definierbaren Zahl n kann die 1 vorbeiziehen.

Gruß, WM
Helmut Richter
2020-03-30 16:59:28 UTC
Permalink
Post by Ganzhinterseher
Post by Me
Aber WMs Gerede von "potentiell unendlichen Mengen" ist wirklich unerträglich.
Es ist *völlig unklar*, was das überhaupt sein soll.
Durch Transpositionen kann die 1 aus
1, 2, 3, ...
2, 3, 4, ..., n-1, 1, n, n+1, n+2, ...
aber nicht an allen. Also muss ein Puffer vorhanden sein. Die Menge der
Endsegmente, die diesen Puffer bildet, besitzt nach dem Wohlordnungssatz
ein niederstes Element m. Aber man kann es nicht definieren, da 1 mit
jeder definierbaren Zahl vertauscht werden kann.
Hier fehlt es an den Definitionen von „Puffer“. Außerdem an der von
„vertauscht werden kann“. Eine Vertauschung wäre ja eine weitere
Transposition, und der Effekt von unendlich vielen Transpositionen ist
undefiniert. Man *kann* sie vernünftig definieren, wenn man will. Getan hast
du das nicht. Bei der nach meinem Geschmack naheliegendsten Definition käme
als Grenzwert die Folge 2, 3, 4, ... ohne die 1 heraus, die allerdings keine
bijetive Abbildung der natürlichen Zahlen ist – im Gegensatz zu allen Gliedern
der Folge. Aber das ist ja normal: schließlich hat niemand bewiesen, dass die
Eigenschaften der Glieder einer Folge auch Eigenschaft eines passend
definierten Grenzwerts sind; das ist ja auch bei anderen Folgen in aller Regel
nicht wahr.

Alle Definitionen, die – meist unausgesprochen – so funktionieren, dass sie
den endlichen Fall enthalten und den unendlichen nur durch Analogie bilden
„und dasselbe genauso unendlich oft“, sind Schrott.

--
Helmut Richter
Ganzhinterseher
2020-03-30 17:55:43 UTC
Permalink
Post by Helmut Richter
Post by Ganzhinterseher
Durch Transpositionen kann die 1 aus
1, 2, 3, ...
2, 3, 4, ..., n-1, 1, n, n+1, n+2, ...
aber nicht an allen. Also muss ein Puffer vorhanden sein. Die Menge der
Endsegmente, die diesen Puffer bildet, besitzt nach dem Wohlordnungssatz
ein niederstes Element m. Aber man kann es nicht definieren, da 1 mit
jeder definierbaren Zahl vertauscht werden kann.
Hier fehlt es an den Definitionen von „Puffer“.
Da gibt es zwei: Reibekuchen ist nicht gemeint.
Post by Helmut Richter
Außerdem an der von
„vertauscht werden kann“. Eine Vertauschung wäre ja eine weitere
Transposition, und der Effekt von unendlich vielen Transpositionen ist
undefiniert.
Der Effekt von unendlich vielen Transpositionen ist genau so definiert oder undefiniert wie der Effekt von unendlich vielen Zuordnungen bei Bijektionen.
Post by Helmut Richter
Man *kann* sie vernünftig definieren, wenn man will. Getan hast
du das nicht.
Man braucht nur die Definition für definierbare n. Dafür gilt mein Beweis. Dafür ist die Vertauschung mit 1 immer möglich..
Post by Helmut Richter
Bei der nach meinem Geschmack naheliegendsten Definition käme
als Grenzwert die Folge 2, 3, 4, ... ohne die 1 heraus
Bei Umordnungen können keine Glieder hinzukommen und keine Glieder verlorengehen.
Post by Helmut Richter
schließlich hat niemand bewiesen, dass die
Eigenschaften der Glieder einer Folge auch Eigenschaft eines passend
definierten Grenzwerts sind
Es geht nicht um Grenzwerte, sondern um surjektive Abbildungen. Das ist ein Unterschied, der leider in der ML verschlampt wurde.
Post by Helmut Richter
das ist ja auch bei anderen Folgen in aller Regel
nicht wahr.
Folgen können Grenzwerte haben, bei Abbildungen wird jedes Element gebraucht.
Post by Helmut Richter
Alle Definitionen, die – meist unausgesprochen – so funktionieren, dass sie
den endlichen Fall enthalten und den unendlichen nur durch Analogie bilden
„und dasselbe genauso unendlich oft“, sind Schrott.
So zum Beispiel die Abzählung abzählbarer Mengen oder die Bildung der Diagonalzahl in einer unendlichen "quadratischen" Matrix.

Gruß, WM
Ganzhinterseher
2020-03-29 16:55:29 UTC
Permalink
Post by Michael Klemm
Post by Ganzhinterseher
Post by Michael Klemm
2b < 3b < 4b < … < 1a ist in Rainers Schreibweise Ord_Z = [2,3,4,...,1], wobei ich die beiden voneinander unabhängigen Bereiche mit b und a bezeichnet und die Ordnung explizit mit "<" angeben habe. Die Unbegriffe "undefinierbar" bzw. "dunkel" habe ich selbstverständlich auch gestrichen.
Tja, du scheinst nicht zu begreifen, dass die Ordnung Z nicht ohne undefinierbare Zahlen aus der Ordnung A erzeugt werden kann und dass ohne die Erzeugbarkeit der Ordnung Z auch keine Surjektivität bei Bijektionen möglich ist.
Wer sagt denn, dass die Ordnung 2b < 3b < 4b < … < 1a aus einer anderen Ordnung erzeugt wird?
Ich sage, dass bei einer festen Menge endlicher Zahlen und endlich nummerierter Plätze jede Ordnung erzeugt werden kann. Zum Beispiel
Ord_Z = 2, 3, 4, ..., 1
kann durch unendlich viele Transpositionen der Form (i, j) zwischen den Plätzen i und j:
(1, 2), (2, 3), 3, 4) ...
aus
Ord_A = 1, 2, 3, 4, ...
erzeugt werden.

Dass tatsächlich alle Transpositionen drankommen, kann man aus der surjektiven Abbildung von |N auf die Menge der Transpositionen beweisen:
n --> (n-1, n).

Wenn das nicht ginge, dann ginge auch die Surjektion von |N auf die Menge der Brüche nicht.

Gruß, WM
Me
2020-03-29 17:15:57 UTC
Permalink
Ich sage [...]
Hast Du Dir schon den Unterschied zwischen unendlichen Folgen und geordneten Mengen klar gemacht?
Ganzhinterseher
2020-03-29 18:01:19 UTC
Permalink
Post by Me
Ich sage [...]
Hast Du Dir schon den Unterschied zwischen unendlichen Folgen und geordneten Mengen klar gemacht?
Es gibt keinen Unterschied zwischen Folgen ohne Wiederholungen und geordneten Mengen der Mächtigkeit ℵo. Nach Cantor ist die geordnete Menge

1/1, 1/2, 2/1, 1/3, 3/1, 1/4, 2/3, 3/2, 4/1, 1/5, ...

eine Reihe, womit er Folge meint.

Übrigens gibt es ebenfalls nach Cantor eine surjektive Abbildung von ℕ auf diese Menge.

Die geordnete Menge (1, 2, 3, ...) kann zur Menge (2, 3, 4, ..., 1) umgeformt werden, denn die Menge ((1, 2), (2, 3), (3, 4), ...) der Transpositionen (i, j) zwischen den Plätzen i und j, die das bewerkstelligen, kann ebenfalls surjektiv von ℕ aus abgebildet werden. Gott der Herr hat sie gezähelet, dass ihm auch nicht eine fehelet an der ganzen großen Zahl. Welches allerdings der Index der 1 ist, das bleibt sein Geheimnis. Wir Sterblichen dürfen es nicht wissen. Cantor weiß es vielleicht schon.

Gruß, WM
Me
2020-03-29 18:37:55 UTC
Permalink
Post by Me
Ich sage [...]
Hast Du Dir schon den Unterschied zwischen unendlichen Folgen und
geordneten Mengen klar gemacht?
Es gibt keinen Unterschied zwischen Folgen ohne Wiederholungen und geordneten Mengen der Mächtigkeit ℵo. Nach Cantor [...]
Sorry, aber in der modernen Mathematik gibt es den. Offenbar ist Dir das nicht bekannt.
Rainer Rosenthal
2020-03-26 22:10:16 UTC
Permalink
Post by Ganzhinterseher
Post by Rainer Rosenthal
Weil die 1 bei der Umordnung 2, 3, 4, ..., 1 von 1, 2, 3, 4, ...
auf einem Platz mit unendlichem Index landet.
Wo ist denn eine solche Umordnung beschrieben?
Oben.
Nein. Da sind zwei Umordnungen Ord_Z und Ord_A *notiert*, und Du
beschreibst, wo die 1 bei fortgesetztes Umordnen von Ord_A zu Ord_Z
schließlich landet. Dieses fortgesetzte Umordnen stellst Du Dir
"irgendwie" vor, aber Du beschreibst es nicht.
Post by Ganzhinterseher
Post by Rainer Rosenthal
Ord_A = [1, 2, 3, 4, ...]
und
Ord_Z = [2, 3, 4, ..., 1]
Dabei ist Ord_A uns allen wohlbekannt, und über Ord_Z hatte ich mich mit
Dir unterhalten können. Dabei bekam ich sogar Zustimmung von Dir, dass
in Ord_Z gilt: das kleinste (niederste) Element in {1,2,3} ist die 2.
Natürlich. Da besteht kein Dissens.
Post by Rainer Rosenthal
Danach ging es leider nicht mehr weiter, und ich habe auf Anraten kluger
Wo ist denn eine Abfolge von Umordnungen beschrieben, die von Ord_A zu
Ord_Z führen kann? Wer behauptet denn, dass das überhaupt möglich sei?
Ich behaupte, ....
Ja, das ist mir inzwischen auch bewusst geworden, dass Du da etwas
behauptest. Du behauptest, dass der Übergang Ord_A ===> Ord_Z möglich
sein muss.
Ich behaupte aber das Gegenteil: Du kannst so lange an Ord_A
Vertauschungen durchführen, wie Du willst, es entstehen stets nur Ord_M
Typen, aber niemals Ord_Z.
Post by Ganzhinterseher
Post by Rainer Rosenthal
Wohin auch immer ich "die 1 defilieren lasse", startend bei Ord_A, komme
ich stets nur zu Ordnungen der Form
Ord_M = [2, 3, 4, ... , 1, ...]
Das liegt daran, dass nur solche Plätze definierbar sind. Denn für alle definierbaren natürlichen Zahlen gilt
∀n ∈ ℕ_def: |ℕ \ {1, 2, 3, ..., n}| = ℵo.
Wie auch immr Du es ausdrücken magst, wir sind uns einig, dass nur Ord_M
erreichbar ist. Du musst mir gar nicht erklären, wovon ich bereits
überzeugt bin.
Post by Ganzhinterseher
Also bleiben stets unendlich viele übrig, die offenbar nicht definierbar sind (sonst würde sie ja aufgebraucht), aber trotzdem natürliche Zahlen sind.
Wie auch immer Du die Situation für Dich erklärst: da ist Konsens, das
nur Ord_M erreichbar ist.
Post by Ganzhinterseher
Post by Rainer Rosenthal
Dein Hütchen-Spiel-Vergleich ist treffend. Angeblich hast Du Cantor
dabei erwischt. Aber wo? Du scheinst ihm eher vorzuwerfen, dass er dies
Hütchenspiel nicht sauber hinbekommen würde.
Wo? Schau Dein Beispiel an: Ord_M = [2, 3, 4, ... , 1, ...]
Oder meines: ∀n ∈ ℕ_def: |ℕ \ {1, 2, 3, ..., n}| = ℵo.
Du wiederholst nur, was Konsens ist.
Post by Ganzhinterseher
Cantor behauptet aber bis zum Ende zu kommen.
Soso, oben sagtest Du noch, das sei eine Behauptung von Dir ("Ich
behaupte ...")
Post by Ganzhinterseher
Das sagt er zwar nicht so deutlich, aber ...
Danke, das wollte ich gerne herausgearbeitet haben.

~~~

Wenn Cantor über Bijektionen zum Mächtigkeits-Vergleich spricht, dann
ist da von "Ordnung" überhaupt keine Rede. Es geht um Kardinalzahlen.
Allerdings hat sich Cantor sehr wohl auch über verschiedene Ordnungen
Gedanken gemacht, was ihn zu den Ordinalzahlen brachte.
Und er hat dabei die "Wohlordnung" erfunden, womit er solche Ordnungen
charakterisiert, in denen jede nicht-leere Teilmenge ein kleinstes
(niederstes) Element hat.

Es wäre schön, wenn Du nach Deinm profunden Literaturstudium zu
Mengenlehre es über Dich bringen könntest, anzuerkennen, dass vor Cantor
kein noch so großer Geist so tief gedacht hat.
Ich sehe da aber schwarz, weil Du selbst lieber vermischst als sauber
trennst.
Me
2020-03-26 23:07:53 UTC
Permalink
Es wäre schön, wenn Du nach Deinem profunden Literaturstudium zur
Mengenlehre es über Dich bringen könntest, anzuerkennen, dass vor Cantor
kein noch so großer Geist so tief gedacht hat.
Das ist jetzt zwar schön gesagt, aber m. E. etwas übertrieben. Wenn wir das aber auf das Gebiet der "Mengenlehre" (im allgemeinsten Sinne) einschränken wollen, muss man Dir sicher Recht geben. Er hat da wirklich Großes geleistet. Man übertreibt wohl nicht, wenn man sagt, dass er ein ganzes (mathematisches) Gebiet "begründet" hat.

Jedoch sollte man an dieser Stelle auch einmal auf einen "Vorläufer" hinweisen: nämlich auf Bolzano und dessen (posthum herausgegebenes) Büchlein "Paradoxien des Unendlichen". Man findet darin schon ein paar sehr interessante Gedanken zum Begriff der "Menge" (und insbesondere auch zu unendlichen Mengen), auch wenn das alles noch s e h r weit von einer mathematischen Theorie entfernt ist.

Siehe dazu auch:
https://glossar.hs-augsburg.de/Menge_(Mengenlehre)
Ganzhinterseher
2020-03-27 19:53:50 UTC
Permalink
Post by Me
Jedoch sollte man an dieser Stelle auch einmal auf einen "Vorläufer" hinweisen: nämlich auf Bolzano und dessen (posthum herausgegebenes) Büchlein "Paradoxien des Unendlichen". Man findet darin schon ein paar sehr interessante Gedanken zum Begriff der "Menge" (und insbesondere auch zu unendlichen Mengen),
Insbesondere die sehr vernünftige Feststellung, dass Bijektionen zwischen unendlichen Mengen nichts über Gleichzahligkeit aussagen.

Gruß, WM
Ganzhinterseher
2020-03-27 19:43:51 UTC
Permalink
Post by Rainer Rosenthal
Nein. Da sind zwei Umordnungen Ord_Z und Ord_A *notiert*, und Du
beschreibst, wo die 1 bei fortgesetztes Umordnen von Ord_A zu Ord_Z
schließlich landet. Dieses fortgesetzte Umordnen stellst Du Dir
"irgendwie" vor, aber Du beschreibst es nicht.
Prinzipiell ist jede Anordnung durch unendlich viele zufallsbasierte Transpositionen erzielbar. Wenn alle natürlichen Zahlen existieren, dann können auch alle vor der 1 existieren. Aber wie die Anordnung auch immer zustandekommt: Die Indexmenge darf nicht wachsen.
Post by Rainer Rosenthal
Ja, das ist mir inzwischen auch bewusst geworden, dass Du da etwas
behauptest. Du behauptest, dass der Übergang Ord_A ===> Ord_Z möglich
sein muss.
Ich behaupte, dass jeder Übergang in jede Ordnung möglich ist, weil alle Zahlen endlich sind und auf endlichen Plätzen stehen.
Post by Rainer Rosenthal
Ich behaupte aber das Gegenteil: Du kannst so lange an Ord_A
Vertauschungen durchführen, wie Du willst, es entstehen stets nur Ord_M
Typen, aber niemals Ord_Z.
Das zeigt, dass nicht alle Zahlen in Ord_A definierbar sind. Alle definierbaren Zahlen können vor die 1 gestellt werden. Das liegt in der Definition der Definierbarkeit. Welche definierbare Zahl würde sich weigern?
Post by Rainer Rosenthal
Post by Ganzhinterseher
Das liegt daran, dass nur solche Plätze definierbar sind. Denn für alle definierbaren natürlichen Zahlen gilt
∀n ∈ ℕ_def: |ℕ \ {1, 2, 3, ..., n}| = ℵo.
Wie auch immer Du es ausdrücken magst, wir sind uns einig, dass nur Ord_M
erreichbar ist. Du musst mir gar nicht erklären, wovon ich bereits
überzeugt bin.
Wenn in Ord_A alle Zahlen vor die 1 gestellt werden können, dann ist Ord_Z erreichbar. Wenn nicht alle Zahlen vor die 1 gestellt werden können, dann muss es eine Grund dafür geben. Dieser Grund ist entweder die Undefinierbarkeit der meisten Zahlen (denn es bleiben ja immer unendlich viele hinter der 1) oder die Nichtexistenz dieser Zahlen. Das würde aber jede Bijektion, zum Beispiel mit Q, ausschließen.
Post by Rainer Rosenthal
Post by Ganzhinterseher
Also bleiben stets unendlich viele übrig, die offenbar nicht definierbar sind (sonst würde sie ja aufgebraucht), aber trotzdem natürliche Zahlen sind.
Wie auch immer Du die Situation für Dich erklärst: da ist Konsens, das
nur Ord_M erreichbar ist.
Warum glaubst Du denn, dass was in Ord_M hinter der 1 steht, zur Nummerierung von Brüchen taugt. Was Brüche nummerierten kann, sollte auch beweglich sein.
Post by Rainer Rosenthal
Post by Ganzhinterseher
Wo? Schau Dein Beispiel an: Ord_M = [2, 3, 4, ... , 1, ...]
Oder meines: ∀n ∈ ℕ_def: |ℕ \ {1, 2, 3, ..., n}| = ℵo.
Du wiederholst nur, was Konsens ist.
Post by Ganzhinterseher
Cantor behauptet aber bis zum Ende zu kommen.
Wenn Cantor über Bijektionen zum Mächtigkeits-Vergleich spricht, dann
ist da von "Ordnung" überhaupt keine Rede.
Da liegst Du leider falsch. Jede natürliche Zahl hat einen Bruch als Partner. Weshalb sonst der Aufwand in
1/1, 1/2, 2/1, 1/3, 2/2, 3/1, 1/4, 2/3, 3/2, 4/1, 1/5, 2/4, 3/3, 4/2, 5/1, 1/6, ... ? Weshalb injektiv und surjektiv?
Post by Rainer Rosenthal
Es geht um Kardinalzahlen.
Allerdings hat sich Cantor sehr wohl auch über verschiedene Ordnungen
Gedanken gemacht, was ihn zu den Ordinalzahlen brachte.
Und er hat dabei die "Wohlordnung" erfunden, womit er solche Ordnungen
charakterisiert, in denen jede nicht-leere Teilmenge ein kleinstes
(niederstes) Element hat.
Merkwürdig, dass die wohlgeordnete Menge der in Ord_M hinter der 1 folgenden Zahlen kein kleinstes Element hat. Findest Du nicht?

Gruß, WM
Juergen Ilse
2020-03-27 21:33:31 UTC
Permalink
Hallo,
Post by Ganzhinterseher
Prinzipiell ist jede Anordnung durch unendlich viele zufallsbasierte
Transpositionen erzielbar.
Das ist so nicht korrekt.
Post by Ganzhinterseher
Wenn alle natürlichen Zahlen existieren, dann können auch alle vor der 1
existieren.
SIE ignorieren, das zwar jede einzelne natuerliche Zahl endlich ist,
es aber *dennoch* unendlich viele diesrr endlichen natuerlichen Zahlen
gibt: es gibt keine "letzte" oder "groesste", da nach jeder beliebigen
noch eine weitere folgt, die ebenfalls wieder endlich ist. Und auch
dieser folgt dann wieder eine weitere (wiederum endliche) natuerliche
Zahl nach. Nur sind SIE unfaehig, das zu begreifen oder gar zu akzep-
tieren.
Post by Ganzhinterseher
Aber wie die Anordnung auch immer zustandekommt: Die Indexmenge darf nicht wachsen.
Es ist *kompletter* *Unfug* bei unendlichen Mengen mit einer "Anzal der
Elemente" oder gar "Gleichzahligkeit" argumentieren zu wollen. Das ist
doch gerade der Grund fuer die Einfuehrung des Begriffs der "Maechtigkeit",
weil das fuer endliche Mengen so praktische "Anzahl der Elemente" bei
unendlichen Mengen einfach nicht mehr passt.
Post by Ganzhinterseher
Post by Rainer Rosenthal
Ja, das ist mir inzwischen auch bewusst geworden, dass Du da etwas
behauptest. Du behauptest, dass der Übergang Ord_A ===> Ord_Z möglich
sein muss.
Ich behaupte, dass jeder Übergang in jede Ordnung möglich ist,
Das ist *nicht* der Fall (zumindest nicht unter Erhalt der "Ordnungs-
isomorphie").
Post by Ganzhinterseher
weil alle Zahlen endlich sind und auf endlichen Plätzen stehen.
... aber dennoch gibt es davon "unendlich viele" und eben keine "groesste"
(oder in deiner verdrehten Argumentation: "es gibt keinen *letzten* Platz).
Bei der zweiten von dir genannten Ordnung gibt es dagegen einen (von der
1 besetzten) *letzten* Platz, oder anders formuliert: in der Ordnung gibt
es ploetzlich eine "groesste Zahl". Die Ordnung ist also "grundsaetzlich
anders" als die "natuerliche Ordnung" der natuerlichen Zahlen. Wenn SIE
nicht gerade Scheuklappen von mindestens der Groesse unserer Galaxie
nutzen, sollte das sogar *IHNEN* auffallen.
Post by Ganzhinterseher
Post by Rainer Rosenthal
Ich behaupte aber das Gegenteil: Du kannst so lange an Ord_A
Vertauschungen durchführen, wie Du willst, es entstehen stets nur Ord_M
Typen, aber niemals Ord_Z.
Das zeigt, dass nicht alle Zahlen in Ord_A definierbar sind.
Nein, das zeigt, dass SIE Mengenlehre noch nicht einmal im Ansatz verstanden
haben, und vermutlich auch unfaehig sind, auch nur irgend etwas an moderner
Mathematik zu begreifen.
Post by Ganzhinterseher
Alle definierbaren Zahlen können vor die 1 gestellt werden. Das liegt
in der Definition der Definierbarkeit.
Das SIE hier bereits behauptet haben, dass eine natuerliche Zahl genau
dann definierbar sei, wenn sie eine Dezimalendarstellung besitzt, sind
*alle* natuerlichen Zahlen definierbar. Der Beweis mittels vollstaendiger
Induktion und dem Algorithmus der "schriftlichen Addition" sollte sogar
fuer SIE hinreichend trivial sein, um zu zeigen, dass der unmittelbare
Nachfolger (+1) einer Zahl mit einer Dezimalendarstellung ebenfalls eine
Dezimalendarstellung besitzt. Zusammen mit der Tatsache, dass die *erste*
natuerliche Zahl (egal, ob man nun die 0 oder die 1 als kleinste bzw.
erste natuerliche Zahl betrachtet) eine Dezimalendarstellung besitzt,
ergibt sich dann, dass *alle* natuerlichen Zahlen "definierbar" sind.
Post by Ganzhinterseher
Welche definierbare Zahl würde sich weigern?
Zahlen koennen sich nicht "weigern", da sie weder einen eigenen Willen haben.

Tschuess,
Juergen Ilse (***@usenet-verwaltung.de)
Ganzhinterseher
2020-03-28 18:58:32 UTC
Permalink
Post by Juergen Ilse
Hallo,
Post by Ganzhinterseher
Prinzipiell ist jede Anordnung durch unendlich viele zufallsbasierte
Transpositionen erzielbar.
Das ist so nicht korrekt.
Wenn alle Zahlen auf endlichen Plätzen sitzen, dann kann jede durch endlich viele Umordnungen vor die 1 gebracht werden. Also können alle durch aleph_0 Umordnungen vor die 1 gebracht werden.
Post by Juergen Ilse
Post by Ganzhinterseher
Wenn alle natürlichen Zahlen existieren, dann können auch alle vor der 1
existieren.
SIE ignorieren, das zwar jede einzelne natuerliche Zahl endlich ist,
es aber *dennoch* unendlich viele diesrr endlichen natuerlichen Zahlen
∀n ∈ ℕ: n*ℵo = ℵo .
Post by Juergen Ilse
es gibt keine "letzte" oder "groesste", da nach jeder beliebigen
noch eine weitere folgt, die ebenfalls wieder endlich ist.
Das sollte man den Abzählern sagen. Die scheinen das nämlich nicht zu wissen und tönen laut von Surjektion.
Post by Juergen Ilse
Und auch
dieser folgt dann wieder eine weitere (wiederum endliche) natuerliche
Zahl nach.
Ja, das ist die potentielle Unendlichkeit. Die Matheologen nehmen indessen an, dass man alle diese Zahlen paaren könnte. Wenn das der Fall wäre, so könnte man auch alle vor die 1 schieben.
Post by Juergen Ilse
Post by Ganzhinterseher
Aber wie die Anordnung auch immer zustandekommt: Die Indexmenge darf nicht wachsen.
Es ist *kompletter* *Unfug* bei unendlichen Mengen mit einer "Anzal der
Elemente" oder gar "Gleichzahligkeit" argumentieren zu wollen.
Das ist eine erfrischend richtige Meinung. Allein die Cantorianer tun es trotzdem.
Post by Juergen Ilse
Das ist
doch gerade der Grund fuer die Einfuehrung des Begriffs der "Maechtigkeit",
weil das fuer endliche Mengen so praktische "Anzahl der Elemente" bei
unendlichen Mengen einfach nicht mehr passt.
Mächtigkeit erfordert surjektive Abbildung. Das ist die Abzählung in Verkleidung.
Post by Juergen Ilse
Post by Ganzhinterseher
Ich behaupte, dass jeder Übergang in jede Ordnung möglich ist,
Das ist *nicht* der Fall (zumindest nicht unter Erhalt der "Ordnungs-
isomorphie").
Dann ist auch die Abzählung nicht möglich.
Post by Juergen Ilse
Post by Ganzhinterseher
weil alle Zahlen endlich sind und auf endlichen Plätzen stehen.
... aber dennoch gibt es davon "unendlich viele"
Aktual unendlich viele, also alle.

Die definierbaren Zahlen sind potentiell unendlich:

In Ord_M = 2, 3, 4, ..., 1, ... können alle definierbaren Zahlen vor die 1 gezogen werden. Beweis: Niemand kann eine definierbare Zahl finden, die das verweigern würde.

Die aktuale Unendlichkeit der Zahlen erfordert, dass jede in einer Bijektion festgelegt ist. Wenn das möglich ist, dann kann auch jede vor die 1 gezogen werden: 2, 3, 4, ..., 1, ohne dass ein Index unendlich wird. Aber nun sind die Vorgänger der 1 nicht mehr definierbar (denn alle definierbaren wurde ja schon in Ord_M vo die 1 gezogen.
Post by Juergen Ilse
Post by Ganzhinterseher
Post by Rainer Rosenthal
Ich behaupte aber das Gegenteil: Du kannst so lange an Ord_A
Vertauschungen durchführen, wie Du willst, es entstehen stets nur Ord_M
Typen, aber niemals Ord_Z.
Das zeigt, dass nicht alle Zahlen in Ord_A definierbar sind.
Nein, das zeigt, dass SIE Mengenlehre noch nicht einmal im Ansatz verstanden
haben
Es zeigt, dass die Matheologen keinen Logik können. Es ist ausgesprochen lächerlich, dass es Experten für Mengenlehre und Logik gibt, jedenfalls nach deren eigenem Urteil.
Post by Juergen Ilse
Post by Ganzhinterseher
Alle definierbaren Zahlen können vor die 1 gestellt werden. Das liegt
in der Definition der Definierbarkeit.
Das SIE hier bereits behauptet haben, dass eine natuerliche Zahl genau
dann definierbar sei, wenn sie eine Dezimalendarstellung besitzt, sind
*alle* natuerlichen Zahlen definierbar.
Das ist falsch. Schon in Deinen Behauptungen zu Endsegmenten kann man das klar erkennen. Definiere nur eine einzige Zahl, deren Endsegment nicht ohne Änderung des Ergebnisses aus

E(1) ∩ E(2) ∩ E(3) ∩ ... = { }

entfernt werden kann. Das kannst Du nicht? Warum wohl? Weil alle Endsegmente definierbarer Zahlen ohne Änderung des Ergebnisses entfernt werden können. Es gibt nämlich eine minimale Menge definierbarer Endsegmente: Sie ist leer.

Gruß, WM
Rainer Rosenthal
2020-03-27 21:39:03 UTC
Permalink
Post by Ganzhinterseher
Merkwürdig, dass die wohlgeordnete Menge der in Ord_M hinter der 1 folgenden Zahlen kein kleinstes Element hat. Findest Du nicht?
Merkwürdig ist, dass Du eine konkrete Anmerkung machst.
Sie ist allerdings falsch, was mich etwas enttäuscht.
Wo bleibt die Sorgfalt?

Nehmen wir doch mal dieses Ord_M: [2,3,4,...,100,1,101,102, ...]
Hier ist die 1 an 2 bis 100 vorbei defiliert.
Die hinter 1 folgenden Zahlen haben 101 als kleinste Zahl.
Was ist da so schwierig?

Gruß,
RR
Ganzhinterseher
2020-03-28 18:14:23 UTC
Permalink
Post by Rainer Rosenthal
Post by Ganzhinterseher
Merkwürdig, dass die wohlgeordnete Menge der in Ord_M hinter der 1 folgenden Zahlen kein kleinstes Element hat. Findest Du nicht?
Merkwürdig ist, dass Du eine konkrete Anmerkung machst.
Sie ist allerdings falsch,
Nein.
Post by Rainer Rosenthal
Nehmen wir doch mal dieses Ord_M: [2,3,4,...,100,1,101,102, ...]
Nein, wir nehmen alle Ord_M. Gibt es eine größte Stelle für die 1? Nein. Trotzdem können niemals alle Nachfolger vorbeidefiliert sein. Das bedeutet, die Menge derer, die definierbar und damit auch defilierbar sind, ist potentiell unendlich. Beweis: Du kannst keine Ord_M annehmen, wo mehr als endliche viele Zahlen vor der 1 stehen. Ursache: Du könntest nicht mehr Zahlen beschreiben, ausdrücken, definieren.
Post by Rainer Rosenthal
Hier ist die 1 an 2 bis 100 vorbei defiliert.
Die hinter 1 folgenden Zahlen haben 101 als kleinste Zahl.
Was ist da so schwierig?
Jede kleinste Zahl, die Du benennen kannst, kann an der 1 vorbeidefilieren. Fast alle sind aber nicht benennbar. *Wenn* sie allerdings in Bijektionen surjektiv erfassbar sind (was im Gegensatz zu Deiner Meinung zur Kardinalitätsbestimmung erforderlich ist), dann sollten sie auch defilieren können. Hier liegt die Inkonsistenz der Mengenlehre.

Gruß, WM
Rainer Rosenthal
2020-03-29 13:29:04 UTC
Permalink
Post by Ganzhinterseher
Post by Rainer Rosenthal
Post by Ganzhinterseher
Merkwürdig, dass die wohlgeordnete Menge der in Ord_M hinter der 1 folgenden Zahlen kein kleinstes Element hat. Findest Du nicht?
Merkwürdig ist, dass Du eine konkrete Anmerkung machst.
Sie ist allerdings falsch,
Nein.
Post by Rainer Rosenthal
Nehmen wir doch mal dieses Ord_M: [2,3,4,...,100,1,101,102, ...]
Nein, wir nehmen alle Ord_M.
Nein, nehmen wir nicht. Es ging um Ord_M, und zwar um irgendeine der
Ordnungen dieses Typs.

Es ist wenig lustig, wenn Du stets was anderes schreibst, als was Du
meinst.
Ganzhinterseher
2020-03-29 16:53:37 UTC
Permalink
Post by Rainer Rosenthal
Post by Ganzhinterseher
Post by Rainer Rosenthal
Post by Ganzhinterseher
Merkwürdig, dass die wohlgeordnete Menge der in Ord_M hinter der 1 folgenden Zahlen kein kleinstes Element hat. Findest Du nicht?
Merkwürdig ist, dass Du eine konkrete Anmerkung machst.
Sie ist allerdings falsch,
Nein.
Post by Rainer Rosenthal
Nehmen wir doch mal dieses Ord_M: [2,3,4,...,100,1,101,102, ...]
Nein, wir nehmen alle Ord_M.
Nein, nehmen wir nicht.
Ich sagte: "Merkwürdig, dass die wohlgeordnete Menge der in Ord_M hinter der 1 folgenden Zahlen kein kleinstes Element hat." Dabei habe ich keine konkrete Ordnung angegeben. Falls Du ursprünglich wirklich nicht verstanden hast, was ich meinte, und mich für blöde genug hältst, die Behauptung für eine konkrete Ordnung zu machen, dann sollte dieser Hinweis wohl genügen.
Post by Rainer Rosenthal
Es ging um Ord_M, und zwar um irgendeine der
Ordnungen dieses Typs.
Nein. Meine Bemerkung bezieht sich auf die Menge aller möglichen Ord_M. Dort ist kein kleinstes auf die 1 folgendes Element auffindbar, obwohl die entsprechende Untermenge der natürlichen Zahlen wohlgeordnet ist.

Gruß, WM
Michael Klemm
2020-03-29 19:08:39 UTC
Permalink
Post by Ganzhinterseher
Post by Rainer Rosenthal
Post by Ganzhinterseher
Post by Rainer Rosenthal
Post by Ganzhinterseher
Merkwürdig, dass die wohlgeordnete Menge der in Ord_M hinter der 1 folgenden Zahlen kein kleinstes Element hat. Findest Du nicht?
Merkwürdig ist, dass Du eine konkrete Anmerkung machst.
Sie ist allerdings falsch,
Nein.
Post by Rainer Rosenthal
Nehmen wir doch mal dieses Ord_M: [2,3,4,...,100,1,101,102, ...]
Nein, wir nehmen alle Ord_M.
Nein, nehmen wir nicht.
Ich sagte: "Merkwürdig, dass die wohlgeordnete Menge der in Ord_M hinter der 1 folgenden Zahlen kein kleinstes Element hat." Dabei habe ich keine konkrete Ordnung angegeben. Falls Du ursprünglich wirklich nicht verstanden hast, was ich meinte, und mich für blöde genug hältst, die Behauptung für eine konkrete Ordnung zu machen, dann sollte dieser Hinweis wohl genügen.
Post by Rainer Rosenthal
Es ging um Ord_M, und zwar um irgendeine der
Ordnungen dieses Typs.
Nein. Meine Bemerkung bezieht sich auf die Menge aller möglichen Ord_M. Dort ist kein kleinstes auf die 1 folgendes Element auffindbar, obwohl die entsprechende Untermenge der natürlichen Zahlen wohlgeordnet ist.
Das ist allerdings kein Wunder, da die 1 kein Element dieser Menge ist.

Gruß
Michael
Ganzhinterseher
2020-03-30 15:07:31 UTC
Permalink
Post by Ganzhinterseher
Meine Bemerkung bezieht sich auf die Menge aller möglichen Ord_M. Dort ist kein kleinstes auf die 1 folgendes Element auffindbar, obwohl die entsprechende Untermenge der natürlichen Zahlen wohlgeordnet ist.
Das ist allerdings kein Wunder, da die 1 kein Element dieser Menge ist.
Es geht um die auf die 1 folgende Menge:

2, 3, 4, ..., n-1, 1, n, n+1, n+2, ...

Wenn die 1 nicht an allen Zahlen vorbeidefilieren kann, dann muss ein Puffer vorhanden sein. Dann besitzt die Menge der Endsegmente, die diesen Puffer bildet, nach dem Wohlordnungssatz ein niederstes Element m.

Natürlich ist das m nicht definierbar, also eine dunkle Zahl. Denn an jeder definierbaren Zahl n kann die 1 vorbeiziehen.

Gruß, WM
Michael Klemm
2020-03-30 15:21:46 UTC
Permalink
Post by Ganzhinterseher
Post by Ganzhinterseher
Meine Bemerkung bezieht sich auf die Menge aller möglichen Ord_M. Dort ist kein kleinstes auf die 1 folgendes Element auffindbar, obwohl die entsprechende Untermenge der natürlichen Zahlen wohlgeordnet ist.
Das ist allerdings kein Wunder, da die 1 kein Element dieser Menge ist.
2, 3, 4, ..., n-1, 1, n, n+1, n+2, ...
Wenn die 1 nicht an allen Zahlen vorbeidefilieren kann, dann muss ein Puffer vorhanden sein. Dann besitzt die Menge der Endsegmente, die diesen Puffer bildet, nach dem Wohlordnungssatz ein niederstes Element m.
Natürlich ist das m nicht definierbar, also eine dunkle Zahl. Denn an jeder definierbaren Zahl n kann die 1 vorbeiziehen.
Gruß, WM
Nein, in dem von Dir Weggeschnittenen, ging es um die Menge von geortneten Mengen {{2,1}, {2,3,1}, {2,3,4,1},...}.

Gruß
Michael
Juergen Ilse
2020-03-31 03:00:09 UTC
Permalink
Hallo,
Post by Ganzhinterseher
2, 3, 4, ..., n-1, 1, n, n+1, n+2, ...
Wenn die 1 nicht an allen Zahlen vorbeidefilieren kann, dann muss ein Puffer vorhanden sein. Dann besitzt die Menge der Endsegmente, die diesen Puffer bildet, nach dem Wohlordnungssatz ein niederstes Element m.
2Mathematisches Bullshit Bingo" hat nichts mit einem Beweis zu tun, SIE
mathematische Nulpe.

Tschuess,
Juergen Ilse (***@usenet-verwaltung.de)
Rainer Rosenthal
2020-03-29 21:58:57 UTC
Permalink
Post by Ganzhinterseher
Merkwürdig, dass die wohlgeordnete Menge der in Ord_M hinter der 1
folgenden Zahlen kein kleinstes Element hat. Findest Du nicht?
Hier ist von einer Ord_M die Rede.
Post by Ganzhinterseher
Nein. Meine Bemerkung bezieht sich auf die Menge aller möglichen Ord_M. Dort ist kein kleinstes auf die 1 folgendes Element auffindbar, obwohl die entsprechende Untermenge der natürlichen Zahlen wohlgeordnet ist.
Na toll, jetzt bezieht sich Deine Aussage plötzlich auf die Menge der
Ord_M. Du musst Dich nicht wundern, wenn Du nicht ernst genommen wirst.

"die wohlgeordnete Menge der in Ord_M hinter der 1 folgenden Zahlen"
muss ich mir also nach einer sich ständig ändernden Bastelanleitung
erstellen, um mich dann mit Dir über ihre interessanten Eigenschaften
wundern zu dürfen. Mathematik sieht anders aus.

Gruß,
RR
Ganzhinterseher
2020-03-30 12:12:10 UTC
Permalink
Post by Ganzhinterseher
Meine Bemerkung bezieht sich auf die Menge aller möglichen Ord_M. Dort ist kein kleinstes auf die 1 folgendes Element auffindbar, obwohl die entsprechende Untermenge der natürlichen Zahlen wohlgeordnet ist.
Na toll, jetzt bezieht sich Deine Aussage plötzlich auf die Menge der
Ord_M.
Nicht plötzlich., sondern schon immer. Wie dem auch sei: Bist Du in der Lage, diese Frage zu beantworten? Oder beschädigt das Dein Cantor-Bild zu sehr?
Post by Ganzhinterseher
"die wohlgeordnete Menge der in Ord_M hinter der 1 folgenden Zahlen"
muss ich mir also nach einer sich ständig ändernden Bastelanleitung
erstellen,
Nein, Du musst lediglich alle Mengen dieser Form betrachten. Bei Unendlichkeitsfreunden sollte das doch nicht auf unüberwindbare Schwierigkeiten stoßen.

Gruß, WM
Rainer Rosenthal
2020-03-30 15:53:18 UTC
Permalink
Post by Ganzhinterseher
Post by Ganzhinterseher
Meine Bemerkung bezieht sich auf die Menge aller möglichen Ord_M. Dort ist kein kleinstes auf die 1 folgendes Element auffindbar, obwohl die entsprechende Untermenge der natürlichen Zahlen wohlgeordnet ist.
Na toll, jetzt bezieht sich Deine Aussage plötzlich auf die Menge der
Ord_M.
Nicht plötzlich., sondern schon immer. Wie dem auch sei: Bist Du in der Lage, diese Frage zu beantworten? Oder beschädigt das Dein Cantor-Bild zu sehr?
Merkwürdig, dass die wohlgeordnete Menge der in Ord_M hinter der 1
folgenden Zahlen kein kleinstes Element hat. Findest Du nicht?
Da war von *einem Ord_M* die Rede. Wenn Du was anderes gemeint hast,
dann hättest Du es schreiben sollen.
Auf dem Weg von Ord_A = [1,2,3,...] zu Ord_Z = [2,3,4,...,1] kommt man
nie um Ziel, wenn man die 1 jeweils um einen Platz vorrücken lässt.
Mit Ord_M = [2,3,4,...,1,...] ist eine Zwischen-Ordnung bezeichnet, bei
der die 1 an m Plätzen vorbeidefiliert ist.
m = 1: Ord_M = [2,1,3,4,...]
m = 2: Ord_M = [2,3,1,4,...]
etc.
m beliebig: Ord_M = [2,3,...m+1,1,...]

"die wohlgeordnete Menge der in Ord_M hinter der 1 folgenden Zahlen"
ist hier für die genannten m die folgende:
m = 1: {3,4,...}
m = 2: {4,5,...}
m beliebig: {m+2,m+3,...}

Gruß,
RR
Ganzhinterseher
2020-03-30 16:18:02 UTC
Permalink
Post by Ganzhinterseher
Post by Ganzhinterseher
Merkwürdig, dass die wohlgeordnete Menge der in Ord_M hinter der 1
folgenden Zahlen kein kleinstes Element hat. Findest Du nicht?
Da war von *einem Ord_M* die Rede.
Es war nicht von einer Ord_M die Rede, sondern von Ord_M. Es gibt ein Ord_A und ein Ord_Z, aber potentiell unendlich viele Ord_M.
Post by Ganzhinterseher
Wenn Du was anderes gemeint hast,
dann hättest Du es schreiben sollen.
Auf dem Weg von Ord_A = [1,2,3,...] zu Ord_Z = [2,3,4,...,1] kommt man
nie um Ziel
Richtig. Aber warum ist das so? An jeder definierbaren Zahl kann die 1 doch vorbeiziehen. Oder wüsstest Du ein Gegenbeispiel?
Post by Ganzhinterseher
, wenn man die 1 jeweils um einen Platz vorrücken lässt.
Mit Ord_M = [2,3,4,...,1,...] ist eine Zwischen-Ordnung bezeichnet, bei
der die 1 an m Plätzen vorbeidefiliert ist.
m = 1: Ord_M = [2,1,3,4,...]
m = 2: Ord_M = [2,3,1,4,...]
etc.
m beliebig: Ord_M = [2,3,...m+1,1,...]
"die wohlgeordnete Menge der in Ord_M hinter der 1 folgenden Zahlen"
m = 1: {3,4,...}
m = 2: {4,5,...}
m beliebig: {m+2,m+3,...}
m ist beliebig, aber nicht ganz beliebig. Unendlich viele Transpositionen sind verboten.

Es gibt keine Bijektion von |N auf die möglichen Transpositionen. Dagegen bleiben bei der Nummerierung aller Brüche keine Brüche und keine natürlichen Zahlen übrig. Ist das nicht etwas einseitig?

Gruß, WM
Me
2020-03-30 16:38:36 UTC
Permalink
Post by Ganzhinterseher
Dagegen bleiben bei der Nummerierung aller Brüche keine Brüche und keine
natürlichen Zahlen übrig. Ist das nicht etwas einseitig?
In der Tat, das ist HÖCHST merkwürdig:

Dass bei der "Nummerierung" ("Abzählung") *aller* Brüche
"keine Brüche und keine natürlichen Zahlen übrig" bleiben.

Sapperlot, w e r hätte d a s gedacht!

Lit.: https://de.wikipedia.org/wiki/Bijektive_Funktion
Rainer Rosenthal
2020-03-30 19:53:39 UTC
Permalink
Post by Ganzhinterseher
Post by Rainer Rosenthal
Post by Ganzhinterseher
Merkwürdig, dass die wohlgeordnete Menge der in Ord_M hinter der 1
folgenden Zahlen kein kleinstes Element hat. Findest Du nicht?
Da war von *einem Ord_M* die Rede.
Es war nicht von einer Ord_M die Rede, sondern von Ord_M. Es gibt ein Ord_A und ein Ord_Z, aber potentiell unendlich viele Ord_M.
Ja, und? Über jedes dieser Ord_M lässt sich eine Aussage formulieren.

Eine korrekte Aussage ist:
die wohlgeordnete Menge der in Ord_M hinter der 1 folgenden Zahlen hat
ein kleinstes Element.

Eine falsche Aussage ist:
die wohlgeordnete Menge der in Ord_M hinter der 1 folgenden Zahlen hat
kein kleinstes Element.

Letztere stammt von Dir, ob Du es nun toll findest oder nicht.
Formuliert als "Merkwürdig, dass die wohlgeordnete Menge der in Ord_M
hinter der 1 folgenden Zahlen kein kleinstes Element hat."

Gruß,
RR
Ganzhinterseher
2020-03-30 20:44:48 UTC
Permalink
Post by Rainer Rosenthal
Post by Ganzhinterseher
Es war nicht von einer Ord_M die Rede, sondern von Ord_M. Es gibt ein Ord_A und ein Ord_Z, aber potentiell unendlich viele Ord_M.
Ja, und? Über jedes dieser Ord_M lässt sich eine Aussage formulieren.
Was ist mit dem Schnitt aller dieser Mengen?

Gruß, WM
Rainer Rosenthal
2020-03-30 22:13:05 UTC
Permalink
Post by Ganzhinterseher
Post by Rainer Rosenthal
Post by Ganzhinterseher
Es war nicht von einer Ord_M die Rede, sondern von Ord_M. Es gibt
ein Ord_A und ein Ord_Z, aber potentiell unendlich viele Ord_M.
Post by Ganzhinterseher
Post by Rainer Rosenthal
Ja, und? Über jedes dieser Ord_M lässt sich eine Aussage formulieren.
Was ist mit dem Schnitt aller dieser Mengen?
Darüber kann man auch Aussagen zu formulieren versuchen.
Du hast die Vorgeschichte mal wieder gelöscht, und da war von Schnitt
Post by Ganzhinterseher
Merkwürdig, dass die wohlgeordnete Menge der in Ord_M hinter der 1
folgenden Zahlen kein kleinstes Element hat. Findest Du nicht?
Ord_M ist dabei irgendeine der Ordnungen [2,3,4,...,m,1,m+1,...].
Und die auf 1 folgenden Zahlen haben m+1 als kleinstes Element.

Also ist da nichts merkwürdig.
Bemerkenswert ist lediglich, dass Du gerne alte Zitate löschst.
In dem zitierten Satz ist keine Rede von Schnitten.

Gruß,
RR
Me
2020-03-30 22:38:50 UTC
Permalink
Post by Rainer Rosenthal
Bemerkenswert ist lediglich, dass Du gerne alte Zitate löschst.
Das ist dem einen oder anderen in sci.math auch schon aufgefallen. Gerne werden auch Fragen oder Einwände weggeschnitten, etc.
Post by Rainer Rosenthal
Ord_M ist dabei irgendeine der Ordnungen [2,3,4,...,m,1,m+1,...].
Und die auf 1 folgenden Zahlen haben m+1 als kleinstes Element.
Selbst an einer Ordnung wie [2, 3, 4, ... 1] wäre AN SICH nichts merkwürdig. Jedoch gibt es eben keine FOLGE, die man so schreiben (definieren) könnte:

(2, 3, 4, ... 1) .

Aus irgend einem Grund macht dieser einfache Sachverhalt Herrn Mückenheim große Schwierigkeiten. (Hinweis @Mückenheim: Es gibt -bezüglich der gewöhnlichen "Ordnung" auf IN- keine größte/letzte natürliche Zahl.)

Vermutlich hat das aber etwas damit zu tun, dass er den Unterschied zwischen einer /geordneten Menge/ und einer /Folge/ entweder nicht kennt, oder nicht begreift.

@Mückenheim: Das hier ist eine geordnete Menge:

(IN, [2, 3, 4, ... 1]) .

Vielleicht könnte man dafür die Schreibweise

{2 < 3 < 4 < ... < 1}

verwenden (wenn man weiß, wofür dieses Symbol steht).

Eine Folge

(2, 3, 4, ... 1) , (*)

gibt es jedoch nicht. Allenfalls die Folge

(2, 3, 4, ... ) .

Oder (unendlich viele) Folgen "der Form"


(2, 3, 4, .. m, 1, m+1, m+2, m+3, ...) ,

mit m e IN, m >= 2.

Zur Erklärung: Wäre (F_n) so eine/diese Folge (*). Dann müsste gelten:

F_1 = 2
F_2 = 3
usw.

Generell: F_n = n+1 für alle n e IN. Ein k e IN mit F_k = 1 gäbe es dann aber nicht. Widerspruch!
Ganzhinterseher
2020-03-31 15:00:13 UTC
Permalink
Post by Me
(IN, [2, 3, 4, ... 1]) .
Nicht jede geordnete Menge ist eine Folge und nicht jede Folge ist eine geordnete Menge.

Aber was Rainer als Ord_M bezeichnet (2, 3, 4, ..., n, 1, n+1, ...), ist eine Folge. Alle diese Folgen unterscheiden sich von Ord_Z = (2, 3, 4, ..., 1). Also müssen sie alle Zahlen enthalten, die nicht vor der 1 stehen. Da die Menge dieser auf 1 folgenden Zahlen wohlgeordnet ist, muss sie ein kleinstes (hier ist die Bezeichnung richtig) Element enthalten, das in allen Mengen Ord_M vorkommt.
Post by Me
Oder (unendlich viele) Folgen "der Form"
(2, 3, 4, .. m, 1, m+1, m+2, m+3, ...) ,
Auch gut. Ob m oder n ist gleichgültig.

Welches ist nun das erste auf 1 folgende Element, das in allen Mengen Ord_M enthalten ist? Oder ist doch ein Ord_Z dabei?

Gruß, WM
Ganzhinterseher
2020-03-31 14:51:12 UTC
Permalink
Post by Ganzhinterseher
Post by Ganzhinterseher
Post by Rainer Rosenthal
Post by Ganzhinterseher
Es war nicht von einer Ord_M die Rede, sondern von Ord_M. Es gibt
ein Ord_A und ein Ord_Z, aber potentiell unendlich viele Ord_M.
Post by Ganzhinterseher
Post by Rainer Rosenthal
Ja, und? Über jedes dieser Ord_M lässt sich eine Aussage formulieren.
Was ist mit dem Schnitt aller dieser Mengen?
Darüber kann man auch Aussagen zu formulieren versuchen.
Du hast die Vorgeschichte mal wieder gelöscht, und da war von Schnitt
Ich habe nach dem minimalen Element gefragt, das in allen Mengen der Ord_M existiert. Denn Du hattest vollmundig behauptete: "... er hat dabei die "Wohlordnung" erfunden, womit er solche Ordnungen charakterisiert, in denen jede nicht-leere Teilmenge ein kleinstes (niederstes) Element hat."

Nun ist ja wohl klar, dass jede Ord_M ein kleinstes Element hat, das auf die 1 folgt. In allen Mengen der Ord_M gibt es solche Elemente, den Ord_M ist nun einmal, nicht Ord_Z. Deswegen habe ich gefragt, welches Element das kleinste ist, das in keiner Ord_M-Menge vor der 1 steht. Wenn jede Ord_M von Ord_Z verschieden ist, dann müsste es ein solches Element geben.
Post by Ganzhinterseher
Post by Ganzhinterseher
Merkwürdig, dass die wohlgeordnete Menge der in Ord_M hinter der 1
folgenden Zahlen kein kleinstes Element hat. Findest Du nicht?
Ord_M ist dabei irgendeine der Ordnungen [2,3,4,...,m,1,m+1,...].
Und die auf 1 folgenden Zahlen haben m+1 als kleinstes Element.
Also ist da nichts merkwürdig.
Nein, das ist so trivial, dass ich gar nicht darauf gekommen wäre, dass Du meine Frage so missverstehen könntest.
Post by Ganzhinterseher
Bemerkenswert ist lediglich, dass Du gerne alte Zitate löschst.
In dem zitierten Satz ist keine Rede von Schnitten.
Ich dachte mir, wenn Du es bisher nicht verstanden hast, dann verstehst Du es vielleicht mit der Erklärung durch Schnitte. Also: Die Menge aller natürlichen Zahlen ist wohlgeordnet. Welche Zahl ist die kleinste, die in allen Ord_M nicht vor der 1 steht? Oder gibt es keine solche Zahl?

Gruß, WM
Uwe Weiss
2020-03-30 15:14:46 UTC
Permalink
Post by Ganzhinterseher
Das liegt daran, dass nur solche Plätze definierbar sind. Denn für alle definierbaren natürlichen Zahlen gilt
...

Ja, die guten, alten, definierbaren natürlichen Zahlen. Wer kennt sie nicht?

Und für alle definierbaren natürlichen Zahlen, die noch nie in eine
Windel geschissen haben, gilt zudem:
Pfefferminzgeschmack ist donnerstags gleich Minussiebentausend.

Und nun?
Me
2020-03-26 17:07:54 UTC
Permalink
Post by Rainer Rosenthal
Wohin auch immer ich "die 1 defilieren lasse", startend bei Ord_A, komme
ich stets nur zu Ordnungen der Form
Ord_M = [2, 3, 4, ... , 1, ...]
Meinst Du damit "Ordnungen der Form"

[2, 3, 4, ... n, 1, n+1, ...]

mit n e IN (und n >= 2)?
Ralf Bader
2020-03-26 18:02:33 UTC
Permalink
Post by Rainer Rosenthal
Weil die 1 bei der Umordnung 2, 3, 4, ..., 1 von 1, 2, 3, 4, ...
auf einem Platz mit unendlichem Index landet.
Wo ist denn eine solche Umordnung beschrieben?
Ord_A = [1, 2, 3, 4, ...]
und
Ord_Z = [2, 3, 4, ..., 1]
Dabei ist Ord_A uns allen wohlbekannt, und über Ord_Z hatte ich mich mit
Dir unterhalten können. Dabei bekam ich sogar Zustimmung von Dir, dass
in Ord_Z gilt: das kleinste (niederste) Element in {1,2,3} ist die 2.
Danach ging es leider nicht mehr weiter, und ich habe auf Anraten kluger
Wo ist denn eine Abfolge von Umordnungen beschrieben, die von Ord_A zu
Ord_Z führen kann? Wer behauptet denn, dass das überhaupt möglich sei?
Wohin auch immer ich "die 1 defilieren lasse", startend bei Ord_A, komme
ich stets nur zu Ordnungen der Form
Ord_M = [2, 3, 4, ... , 1, ...]
Dein Hütchen-Spiel-Vergleich ist treffend. Angeblich hast Du Cantor
dabei erwischt. Aber wo? Du scheinst ihm eher vorzuwerfen, dass er dies
Hütchenspiel nicht sauber hinbekommen würde.
Gruß,
Rainer Rosenthal
Der Hütchen-Spiel-Vergleich ist Blödsinn, und Mückenheim hat Cantor bei
garnix erwischt, denn er ist in der Regel nicht fähig, zu verstehen, was
da steht.

Es gibt zwei Möglichkeiten:

1 - Du läßt Dich (zum Zwecke der Eindeutigkeit) auf ZFC ein, und zwar
konsequent. Du nimmst zur Kenntnis, daß eine Ordnunmgsrelation auf einer
Menge M eine Teilmenge des cartesischen Produktes MxM ist, daß
Abbildungen zwischen Mengen auf diesen gegebene Ordnungen respektieren
oder nicht, daß es insbesondere Ordnungsisomorphismen gibt, daß jede
Wohlordnung ordnungsisomorph zu genau einer Ordinalzahl ist, daß in den
beiden vorliegenden Ordnungen diese Ordinalzahlen verschieden sind usw.

2 - Du eierst mit metaphorischem Gerede über vorbeidefilierende Zahlen,
Dreipünktchenausdrücken und dergleichen herum und versinkst in
Paradoxien des Unendlichen.
Ganzhinterseher
2020-03-26 19:23:44 UTC
Permalink
Post by Ralf Bader
1 - Du läßt Dich (zum Zwecke der Eindeutigkeit) auf ZFC ein, und zwar
konsequent.
Um blöd zu sein und blöd zu bleiben.
Post by Ralf Bader
Du nimmst zur Kenntnis, daß eine Ordnunmgsrelation auf einer
Menge M eine Teilmenge des cartesischen Produktes MxM ist,
oder Du erkennst, dass ausgehend von der Bijektion zwischen natürlichen Zahlen und Plätzen in 1, 2, 3, ... jede Umordnung möglich ist (weil nur endliche Zahlen und Plätze vorhanden sind) ohne eine unendliche Zahl oder einen unendlich indizierten Platz einzuführen.

Das führt zu der Erkenntnis, dass die Behauptung, eine Bijektion sei möglich, aber nur wenn der Autor clever genug ist, einfach Schwachsinn ist. Aber wer sich darauf einlässt, muss das eben ertragen. Die meisten merken es wohl gar nicht.

Gruß, WM
Lesen Sie weiter auf narkive:
Loading...